You are on page 1of 70

Total Marks : 200

Online Prelims TEST - 9 (SUBJECT WISE)


( InsightsIAS Mock Test Series for UPSC Preliminary Exam 2020 ) Mark Scored : 76

1 Consider the following statements regarding the Election Commission of India


1. The conditions of service and tenure of office of the election commissioners generally decided by
Central government
2. It conducts the elections to Parliament, Panchayats and Office of Vice President.
3. Representation of People act, 1951 fixes the term of the members of the Election commission.

Which of the statements given above is/are correct?


A. 1 only
B. 2 and 3 only
C. 1 and 3 only
D. None

Your Answer : D
Correct Answer : D

34
Answer Justification :
0
The conditions of service and tenure of office of the election commissioners6 41 and the
regional commissioners shall be determined by the president. 7 54 statement 1 is
Hence,
incorrect. -8
. c om
Article 324 of the constitution provides that the power
a il of superintendence, direction and
gm
control of elections to parliament, state legislatures,
@vested
the office of president of India and
the office of vice-president of India shall2 be in the election commission. Thus, the
Election Commission is an all-India bodym in2the sense that it is common to both the Central
a
up Hence, statement 2 is incorrect.
government and the state governments.
n
a sa
Representation of People
- d act, 1951 doesn’t fix the term of the members of the Election
commission. Hence,as statement 3 is incorrect.
D
p am
nu to the Union Public Service Commission (UPSC), consider the following statements:
2 With reference
A
1. The constitution of India has not specified the strength of the Commission.
2. The Constitution authorizes the parliament to determine the conditions of service of the chairman
and other members of the Commission.
3. One-half of the members of the Commission should be such persons who have held office for at least
ten years either under the Government of India or under the government of a state.

Which of the statements given above is/are correct?


A. 1 only
B. 1 and 3 only
C. 2 and 3 only
D. 1, 2 and 3

Your Answer : D
Correct Answer : B

Answer Justification :

prelims.insightsonindia.com 1
© Insights Active Learning | All rights reserved - 131815. You may not reproduce, distribute or exploit the contents in any form without
written permission by copyright owner. Copyright infringers may face civil and criminal liability
Total Marks : 200
Online Prelims TEST - 9 (SUBJECT WISE)
( InsightsIAS Mock Test Series for UPSC Preliminary Exam 2020 ) Mark Scored : 76

The UPSC consists of a chairman and other members appointed by the president of India. The
Constitution, without specifying the strength of the Commission has left the matter to the discretion
of the president, who determines its composition. Hence, statement 1 is correct.

Further, no qualifications are prescribed for the Commission’s membership except that one-half of
the members of the Commission should be such persons who have held office for at least ten years
either under the Government of India or under the government of a state. Hence, statement 3 is
correct.

The Constitution also authorises the president to determine the conditions of service of the
chairman and other members of the Commission. Hence, statement 2 is incorrect.

3 With reference to the Finance Commission, consider the following statements:


1. It is a quasi-judicial body.
2. The Constitution authorizes the Parliament to determine the qualifications of members of the
commission and the manner in which they should be selected.
0 3for4 funding of
3. Fifteenth Finance Commission shall also examine whether a separate mechanism
6 41
defence and internal security ought to be set up. 4 5
7
Which of the statements given above is/are correct? -8
A. 1 and 2 only . c om
B. 1 and 3 only a il
C. 2 and 3 only
@ gm
D. 1, 2 and 3 22 m
u pa
Your Answer : C
a n
s
Correct Answer : D
da
a s-
D :
Answer Justification
m
u paof the Constitution of India provides for a Finance
Article 280
An
Commission as a quasi-judicial body. It is constituted by the president
of India every fifth year or at such earlier time as he considers necessary. Hence, statement 1 is
correct.

The Finance Commission consists of a chairman and four other members to be appointed by the
president. They hold office for such period as specified by the president in his order. The
Constitution authorises the Parliament to determine the qualifications of members of the
commission and the manner in which they should be selected. Accordingly, the Parliament
has specified the qualifications of the chairman and members of the commission. Hence,
statement 2 is correct.

The Union Cabinet chaired by Prime Minister Narendra Modi has approved the proposed
amendment to enable Fifteenth Finance Commission to address serious concerns regarding the
allocation of adequate, secure and non-lapsable funds for defence and internal security of India.

prelims.insightsonindia.com 2
© Insights Active Learning | All rights reserved - 131815. You may not reproduce, distribute or exploit the contents in any form without
written permission by copyright owner. Copyright infringers may face civil and criminal liability
Total Marks : 200
Online Prelims TEST - 9 (SUBJECT WISE)
( InsightsIAS Mock Test Series for UPSC Preliminary Exam 2020 ) Mark Scored : 76

The amendment provides that Fifteenth Finance Commission shall also examine whether a
separate mechanism for funding of defence and internal security ought to be set up and if
so how such a mechanism could be operationalized. Hence, statement 3 is correct.

https://www.pmindia.gov.in/en/news_updates/cabinet-approves-amendment-in-the-terms-of-referenc
e-for-the-fifteenth-finance-commission/

4 Consider the following statements


1. A service voter may appoint any person irrespective of his residency as his / her proxy to give vote
on his / her behalf and in his / her name at the polling station.
2. The facility of service voter is available only to the wife of a male service voter and is not available
to the husband of a female service voter.

Which of the statements given above is/are correct?


A. 1 only
4
03
B. 2 only
1
64
C. Both 1 and 2
D. Neither 1 nor 2
5 4
- 87
om
Your Answer : A
Correct Answer : B
i l .c
m a
g
2@
Answer Justification :
2
Who is a ‘proxy’? am
n up
A service voter may appoint a sa(by applying to Returning Officer in Form 13 F of Conduct of
d
Elections Rules, 1961 – -Form available at the website of Election Commission) any person as his /
s
a on his / her behalf and in his / her name at the polling station. The
her proxy to give Dvote
m to be ordinary resident of that constituency. He need not be a registered
proxy shallahave
p
voter butuhe / she must not be disqualified to be registered as a voter. Hence, statement 1 is
An
incorrect.

Under the existing law, this facility is available only to the wife of a male service voter and
is not available to the husband of a female service voter.

Hence, statement 2 is correct.

https://eci.gov.in/faqs/voter-electors/service-voter/faqs-service-electors-r13/

5 Consider the following statements:


1. The entire expenses including the salaries, allowances and pensions of the chairman and members
of the UPSC are subjected to vote of Parliament.
2. The conditions of service of the chairman or a member cannot be varied to his disadvantage after
his appointment.
3. The jurisdiction of UPSC can be extended by the Parliament.

prelims.insightsonindia.com 3
© Insights Active Learning | All rights reserved - 131815. You may not reproduce, distribute or exploit the contents in any form without
written permission by copyright owner. Copyright infringers may face civil and criminal liability
Total Marks : 200
Online Prelims TEST - 9 (SUBJECT WISE)
( InsightsIAS Mock Test Series for UPSC Preliminary Exam 2020 ) Mark Scored : 76

Which of the statements given above is/are correct?


A. 1 and 2 only
B. 2 and 3 only
C. 3 only
D. 1, 2 and 3

Your Answer : B
Correct Answer : B

Answer Justification :

The entire expenses including the salaries, allowances and pensions of the chairman and members
of the UPSC are charged on the Consolidated Fund of India. Thus, they are not subject to vote of
Parliament. Hence, statement 1 is incorrect.

The conditions of service of the chairman or a member, though determined by the president, cannot
34
be varied to his disadvantage after his appointment. Hence, statement 2 is correct.
0
6 41
4 on UPSC by the
The additional functions relating to the services of the Union can be conferred
5
7
Parliament. It can also place the personnel system of any authority,8corporate body or public
- of UPSC can be extended
m
institution within the jurisdiction of the UPSC. Thus, the jurisdiction
by an act made by the Parliament. Hence, statement
i l .c3ois correct.
g ma
6 With reference to the Finance Commission, consider
2 2@ the following statements:
1. The recommendations made by the Finance
p am Commission are binding on the government.
u
an
2. The commission submits its report to the union Finance minister.
s
da above is/are correct?
Which of the statements- given
s
A. 1 only Da
B. 2 only m
u
C. Both p1aand 2
n
D.ANeither 1 nor 2

Your Answer : D
Correct Answer : D

Answer Justification :

Article 280 of the Constitution of India provides for a Finance Commission as a quasi-judicial body.
It is constituted by the president of India every fifth year or at such earlier time as he considers
necessary.

The commission submits its report to the president. He lays it before both the Houses of
Parliament along with an explanatory memorandum as to the action taken on its recommendations.
Hence, statement 2 is incorrect.

The recommendations made by the Finance Commission are only of advisory nature and hence, not
binding on the government. It is up to the Union government to implement its recommendations on

prelims.insightsonindia.com 4
© Insights Active Learning | All rights reserved - 131815. You may not reproduce, distribute or exploit the contents in any form without
written permission by copyright owner. Copyright infringers may face civil and criminal liability
Total Marks : 200
Online Prelims TEST - 9 (SUBJECT WISE)
( InsightsIAS Mock Test Series for UPSC Preliminary Exam 2020 ) Mark Scored : 76

granting money to the states. Hence, statement 1 is incorrect.

7 With reference to the State Public Service Commission (SPSC), consider the following statements:
1. The chairman and members of the Commission hold office for a term of five years or until they
attain the age of 62 years.
2. The chairman and members of a SPSC can be removed by the governor of the concerned state.

Which of the statements given above is/are correct?


A. 1 only
B. 2 only
C. Both 1 and 2
D. Neither 1 nor 2

Your Answer : A
Correct Answer : D
4
1 03
Answer Justification :
64
54
The chairman and members of the Commission hold office for a term
- 87of six years or until they attain
o m
the age of 62 years, whichever is earlier. However, they can relinquish their offices at any time by
l. c
addressing their resignation to the governor. Hence, statement 1 is incorrect.
i
m a
Although the chairman and members of a SPSC are
@ g appointed by the governor, they can be
2
2 can remove them on the same grounds and in the
removed only by the president. The president
m
pa or a member of the UPSC. Hence, statement 2 is
same manner as he can remove a chairman
u
incorrect. n a
s
- da
8 Consider the following a s
D statements regarding contesting an election in India
m registered as a voter in any of the constituency in India can also contest in
1. If a person is not
a
election.u p
An is registered voter in Delhi, he/she contest an election to Lok Sabha from any
2. If a person
constituency in the country.
3. A person confined in jail can also vote in an election.

Which of the statements given above is/are correct?


A. 1 and 2 only
B. 2 only
C. 2 and 3 only
D. None

Your Answer : B
Correct Answer : D

Answer Justification :

All the above statements are incorrect.

prelims.insightsonindia.com 5
© Insights Active Learning | All rights reserved - 131815. You may not reproduce, distribute or exploit the contents in any form without
written permission by copyright owner. Copyright infringers may face civil and criminal liability
Total Marks : 200
Online Prelims TEST - 9 (SUBJECT WISE)
( InsightsIAS Mock Test Series for UPSC Preliminary Exam 2020 ) Mark Scored : 76

If I am not registered as a voter in any Constituency, can I contest election?

Ans. No

For contesting an election as a candidate a person must be registered as a voter. Sec 4 (d)
of Representation People Act, 1951 precludes a person from contesting unless he is an
elector in any parliamentary constituency. Section 5 (c) of R. P. Act, 1951 has a similar
provision for Assembly Constituencies. Hence, statement 1 is incorrect.

If you are a registered voter in Delhi, you can contest an election to Lok Sabha from any
constituency in the country except Assam, Lakshadweep and Sikkim, as per Section 4 (c), 4
(cc) and 4 (ccc) of the R. P. Act, 1951. Hence, statement 2 is incorrect.

Can a person confined in jail vote in an election?

Ans. No
4
According to section 62(5) of the Representation of the People Act, 1951,
1 03no person shall
vote at any election if he is confined in a prison, whether under a sentence
4 64 of imprisonment or
transportation or otherwise, or is in the lawful custody of the police. 7 5 statement 3 is
Hence,
incorrect. - 8
. c om
il
https://eci.gov.in/faqs/elections/contesting-for-elections/faqs-contesting-for-elections-r4/
a
@ gm
9 Consider the following statements: 2 2
m
1. Joint State Public Service Commission
u pa(JSPSC) is a constitutional body.
2. Joint Public Service Commission
s anwas provided by the Government of India Act of 1919.
3. A JSPSC presents its annual a
d performance report to each of the concerned state governors.
s -
Da given above is/are correct?
Which of the statements
p
A. 3 only am
u 3 only
B. 2nand
A
C. 1 only
D. None

Your Answer :
Correct Answer : A

Answer Justification :

The Constitution makes a provision for the establishment of a Joint State Public Service Commission
(JSPSC) for two or more states. While the UPSC and the SPSC are created directly by the
Constitution, a JSPSC can be created by an act of Parliament on the request of the state legislatures
concerned. Thus, a JSPSC is a statutory and not a constitutional body. Hence, statement 1
is incorrect.

A JSPSC presents its annual performance report to each of the concerned state governors. Each
governor places the report before the state legislature. Hence, statement 3 is correct.

prelims.insightsonindia.com 6
© Insights Active Learning | All rights reserved - 131815. You may not reproduce, distribute or exploit the contents in any form without
written permission by copyright owner. Copyright infringers may face civil and criminal liability
Total Marks : 200
Online Prelims TEST - 9 (SUBJECT WISE)
( InsightsIAS Mock Test Series for UPSC Preliminary Exam 2020 ) Mark Scored : 76

As provided by the Government of India Act of 1919, a Central Public Service Commission was set
up in 1926 and entrusted with the task of recruiting civil servants. The Government of India Act
of 1935 provided for the establishment of not only a Federal Public Service Commission
but also a Provincial Public Service Commission and Joint Public Service Commission for
two or more provinces. Hence, statement 2 is incorrect.

10 Which of the following statements is/are the functions and powers of the National Commission for
Scheduled Castes?
1. To inquire into specific complaints with respect to the deprivation of rights and safeguards of the
SCs
2. To participate and advise on the planning process of socio-economic development of the SCs and to
evaluate the progress of their development under the Union or a state
3. The Commission has to investigate all matters relating to the constitutional and other legal
safeguards for the OBCs and the Anglo-Indian Community and report to the President upon their
working.
4
1 03
64
Select the correct answer using the code given below.
A. 1 and 2 only
5 4
B. 1 only
- 87
om
C. 2 and 3 only
D. 1, 2 and 3
i l .c
m a
g
Your Answer : A
Correct Answer : D 2 2@
am
up
Answer Justification :
s an
da
- Scheduled Castes (SCs) is a constitutional body in the sense that it is
National Commissions for
a
D by Article 338 of the Constitution
directly established
p am
u
The functions of the Commission are:
An
(a) To investigate and monitor all matters relating to the constitutional and other legal safeguards
for the SCs and to evaluate their working;

(b) To inquire into specific complaints with respect to the deprivation of rights and
safeguards of the SCs; Hence, statement 1 is correct.

(c) To participate and advise on the planning process of socio-economic development of


the SCs and to evaluate the progress of their development under the Union or a state;
Hence, statement 2 is correct.

(d) To present to the President, annually and at such other times as it may deem fit, reports upon
the working of those safeguards;

(e) To make recommendations as to the measures that should be taken by the Union or a state for
the effective implementation of those safeguards and other measures for the protection, welfare and
socio-economic development of the SCs; and

prelims.insightsonindia.com 7
© Insights Active Learning | All rights reserved - 131815. You may not reproduce, distribute or exploit the contents in any form without
written permission by copyright owner. Copyright infringers may face civil and criminal liability
Total Marks : 200
Online Prelims TEST - 9 (SUBJECT WISE)
( InsightsIAS Mock Test Series for UPSC Preliminary Exam 2020 ) Mark Scored : 76

(f) To discharge such other functions in relation to the protection, welfare and development and
advancement of the SCs as the president may specify.

The Commission is also required to discharge similar functions with regard to the other
backward classes (OBCs) and the Anglo-Indian Community as it does with respect to the
SCs. In other words, the Commission has to investigate all matters relating to the constitutional and
other legal safeguards for the OBCs and the Anglo-Indian Community and report to the President
upon their working. Hence, statement 3 is correct.

Clause 10 of Article 338 reads as follows: “In this article, references to the Scheduled
Castes shall be construed as including references to such other backward classes as the
President may, on receipt of the report of a Commission appointed under clause (1) of
article 340 by order specify and also to the Anglo-Indian Community”

11 Which of the following is/are the functions of National Human Rights Commission (NHRC):
1. To visit jails and detention places to study the living conditions of inmates and make4
recommendation thereon. 1 03
2. To review the factors including acts of terrorism that inhibit the enjoyment4 6of4 human rights and
recommend remedial measures. 8 75
3. To inquire into any violation of human rights or negligence in the-prevention of such violation by a
public servant, however, only on a petition presented to it.corom
on an order of a court but not suo
i l
maon human rights and make recommendations
motu.
g
4. To study treaties and other international instruments
@
for their effective implementation. 22
p am
Select the correct answer using n u
the codes given below
s a
A. 1, 2 and 3 only
B. 1, 3 and 4 only -
da
s
Da
C. 1, 2 and 4 only
m4
D. 1, 2, 3aand
p
A nu
Your Answer : C
Correct Answer : C

Answer Justification :

The National Human Rights Commission is a statutory but non-constitutional body. It was
established in 1993 under a legislation enacted by the Parliament, namely, the Protection of
Human Rights Act, 1993

The functions of the National Human Rights Commission (NHRC) are:

1. To inquire into any violation of human rights or negligence in the prevention of such violation by
a public servant, either suo motu or on a petition presented to it or on an order of a court.
Hence, statement 3 is incorrect.

2. To intervene in any proceeding involving allegation of violation of human rights pending before a
court.

prelims.insightsonindia.com 8
© Insights Active Learning | All rights reserved - 131815. You may not reproduce, distribute or exploit the contents in any form without
written permission by copyright owner. Copyright infringers may face civil and criminal liability
Total Marks : 200
Online Prelims TEST - 9 (SUBJECT WISE)
( InsightsIAS Mock Test Series for UPSC Preliminary Exam 2020 ) Mark Scored : 76

3. To visit jails and detention places to study the living conditions of inmates and make
recommendation thereon. Hence, statement 1 is correct.

4. To review the constitutional and other legal safeguards for the protection of human rights and
recommend measures for their effective implementation.

5. To review the factors including acts of terrorism that inhibit the enjoyment of human
rights and recommend remedial measures. Hence, statement 2 is correct.

6. To study treaties and other international instruments on human rights and make
recommendations for their effective implementation. Hence, statement 4 is correct.

12 Which of the following Constitutional Amendment Act bifurcated the National Commission for SCs
and STs and created a separate National Commission for Scheduled Tribes?

A. 65th Constitutional Amendment Act 4


B. 85th Constitutional Amendment Act
1 03
C. 69th Constitutional Amendment Act
4 64
5
D. 89th Constitutional Amendment Act
- 87
Your Answer :
.com
Correct Answer : D a i l
g m
Answer Justification : 2@
a m2
The National Commission for SCs u p STs came into being consequent upon passing of the 65th
and
n
Constitutional Amendment Act
a saof 1990. The Commission was established under Article 338 of the
- d of monitoring all the safeguards provided for the SCs and STs
Constitution with the objective
s
Da
under the Constitution or other laws.

am
In order topsafeguard the interests of the STs more effectively, it was proposed to set up a separate
u
An Commission for STs by bifurcating the existing combined National Commission for SCs and
National
STs. This was done by passing the 89th Constitutional Amendment Act of 2003. This Act further
amended Article 338 and inserted a new Article 338-A in the Constitution.

Hence, option (d) is correct.

13 Which of the following schedule was added by 52nd Constitutional Amendment Act?

A. Seventh Schedule
B. Ninth Schedule
C. Tenth Schedule
D. Eleventh Schedule

Your Answer : C
Correct Answer : C

prelims.insightsonindia.com 9
© Insights Active Learning | All rights reserved - 131815. You may not reproduce, distribute or exploit the contents in any form without
written permission by copyright owner. Copyright infringers may face civil and criminal liability
Total Marks : 200
Online Prelims TEST - 9 (SUBJECT WISE)
( InsightsIAS Mock Test Series for UPSC Preliminary Exam 2020 ) Mark Scored : 76

Answer Justification :

The 52nd Amendment Act of 1985 provided for the disqualification of the members of
Parliament and the state legislatures on the ground of defection from one political party to another.
For this purpose, it made changes in four Articles of the Constitution and added a new Schedule
(the Tenth Schedule) to the Constitution. This act is often referred to as the ‘anti-defection
law’. Hence, option (c) is correct.

14 With reference to the Chief Election Commissioner, consider the following statements:
1. He/she hold his/her office till the pleasure of the president.
2. The service conditions of the chief election commissioner cannot be varied to his disadvantage after
his appointment.

Which of the statements given above is/are correct?


A. 1 only
4
03
B. 2 only
1
64
C. Both 1 and 2
D. Neither 1 nor 2
5 4
- 87
om
Your Answer : B
Correct Answer : B
i l .c
m a
g
2@
Answer Justification :
2
am with the security of tenure. He cannot be removed from
The chief election commissioner is provided
p
u
his office except in same mannernand on the same grounds as a judge of the Supreme Court. Thus,
a the pleasure of the president, though he is appointed by him.
still
he does not hold his office
d a
Hence, statement 1 is-incorrect.
s
Da
p am
The service conditions of the chief election commissioner cannot be varied to his disadvantage after
u
his appointment. Hence, statement 2 is correct.
An
15 With reference to Central Bureau of Investigation (CBI), consider the following statements:
1. The CBI is a statutory body.
2. It derives its powers from the Delhi Special Police Establishment Act, 1946.

Which of the statements given above is/are correct?


A. 1 only
B. 2 only
C. Both 1 and 2
D. Neither 1 nor 2

Your Answer : B
Correct Answer : B

Answer Justification :

prelims.insightsonindia.com 10
© Insights Active Learning | All rights reserved - 131815. You may not reproduce, distribute or exploit the contents in any form without
written permission by copyright owner. Copyright infringers may face civil and criminal liability
Total Marks : 200
Online Prelims TEST - 9 (SUBJECT WISE)
( InsightsIAS Mock Test Series for UPSC Preliminary Exam 2020 ) Mark Scored : 76

The CBI is the main investigating agency of the Central Government.

The establishment of the CBI was recommended by the Santhanam Committee on Prevention of
Corruption (1962-1964). The CBI is not a statutory body. Hence, statement 1 is incorrect.

It derives its powers from the Delhi Special Police Establishment Act, 1946. Hence,
statement 2 is correct.

16 With reference to NITI Aayog, consider the following statements:


1. It is a non-statutory body.
2. President of India is the chairperson of NITI Aayog.
3. Vice Chairperson of NITI Aayog enjoys the rank of cabinet minister.

Which of the statements given above is/are correct?


A. 1 only
B. 2 only 4
C. 1 and 3 only 1 03
D. 1, 2 and 3 4 64
5
- 87
Your Answer : C
Correct Answer : C .com
a i l
Answer Justification : g m
2 2@
amCommission, was also created by an executive
The NITI Aayog, like that of the Planning
p
u
resolution of the Governmentnof India (i.e., Union Cabinet). Hence, it is also neither a
sa
constitutional body nor aastatutory body. Hence, statement 1 is correct.

s -d
The Prime Minister a
D of India is the chairperson of NITI Aayog. Hence, statement 2 is incorrect.
p am of NITI Aayog is appointed by the Prime Minister. He enjoys the rank of a
Vice-Chairperson
u
An Minister. Hence, statement 3 is correct.
Cabinet

17 With reference to the National Commission for Scheduled Castes (SCs), consider the following
statements:
1. It is directly established by Article 338 of the Constitution.
2. The Commission, while investigating any matter or inquiring into any complaint, has all the powers
of a criminal court trying a suit.
3. It can submit a report to President as and when it thinks necessary.

Which of the statements given above is/are correct?


A. 1 and 2 only
B. 3 only
C. 1 and 3 only
D. 1, 2 and 3

Your Answer : D

prelims.insightsonindia.com 11
© Insights Active Learning | All rights reserved - 131815. You may not reproduce, distribute or exploit the contents in any form without
written permission by copyright owner. Copyright infringers may face civil and criminal liability
Total Marks : 200
Online Prelims TEST - 9 (SUBJECT WISE)
( InsightsIAS Mock Test Series for UPSC Preliminary Exam 2020 ) Mark Scored : 76

Correct Answer : C

Answer Justification :

The National Commission for Scheduled Castes (SCs) is a constitutional body in the sense that it is
directly established by Article 338 of the Constitution. Hence, statement 1 is correct.

The Commission, while investigating any matter or inquiring into any complaint, has all the powers
of a civil court trying a suit. Hence, statement 2 is incorrect.

The commission presents an annual report to the president. It can also submit a report as
and when it thinks necessary. Hence, statement 3 is correct. The President places all such
reports before the Parliament, along with a memorandum explaining the action taken on the
recommendations made by the Commission. The memorandum should also contain the reasons for
the
non-acceptance of any of such recommendations.

0 34
1
4statements:
18 With reference to the Central Vigilance Commissioner, consider the following
4 6
1. Central Vigilance Commissioner appointed by the President of India.75
- 8for further employment under
2. After his/her tenure, Central Vigilance Commissioner is not eligible
the Central or a state government.
. c om
il the Central Vigilance Commissioner are
3. The salary, allowances and other conditions of serviceaof
similar to those of the Chairman of UPSC
@ gm
2 2
m correct?
pa
Which of the statements given above is/are
A. 1 and 2 only
n u
B. 2 only
a sa
C. 3 only
s -d
D. 1, 2 and 3 Da

p am
u :D
Your Answer
n
A
Correct Answer : D

Answer Justification :

The CVC is a multi-member body consisting of a Central Vigilance Commissioner (chairperson) and
not more than two vigilance commissioners.
They are appointed by the president by warrant under his hand and seal on the
recommendation of a three-member committee consisting of the prime minister as its
head, the Union minister of home affairs and the Leader of the Opposition in the Lok
Sabha. Hence, statement 1 is correct.

They hold office for a term of four years or until they attain the age of sixty five years, whichever is
earlier. After their tenure, they are not eligible for further employment under the Central
or a state government. Hence, statement 2 is correct.

The salary, allowances and other conditions of service of the Central Vigilance
Commissioner are similar to those of the Chairman of UPSC and that of the vigilance

prelims.insightsonindia.com 12
© Insights Active Learning | All rights reserved - 131815. You may not reproduce, distribute or exploit the contents in any form without
written permission by copyright owner. Copyright infringers may face civil and criminal liability
Total Marks : 200
Online Prelims TEST - 9 (SUBJECT WISE)
( InsightsIAS Mock Test Series for UPSC Preliminary Exam 2020 ) Mark Scored : 76

commissioner are similar to those of a member of UPSC.


But they cannot be varied to his disadvantage after his appointment. Hence, statement 3 is
correct.

19 Which of the following bodies have the leader of the opposition in Rajya Sabha, in their appointment
committees?

A. The National Human Rights Commission


B. The Central Information Commission
C. The Central Information Commission
D. CEO of NITI Aayog

Your Answer : A
Correct Answer : A

Answer Justification :
0 34
6 41
5 4
The National Human Rights Commission is a statutory (and not a constitutional) body. It was
established in 1993 under a legislation enacted by the Parliament, 8 7
namely, the Protection of Human
Rights Act, 1993. -
. c om
The chairman and members are appointed by the a il
president on the recommendations of a six-
m
member committee consisting of the primegminister as its head, the Speaker of the Lok
Sabha, the Deputy Chairman of the Rajya 2 2@ Sabha, leaders of the Opposition in both the
m
Houses of Parliament and the Central
u pa home minister.
n
sa consisting of a Central Vigilance Commissioner (chairperson) and
The CVC is a multi-member body
a
- d commissioners. They are appointed by the president by warrant
not more than two vigilance
s
under his hand and
Da seal on the recommendation of a three-member committee consisting
of the primem minister as its head, the Union minister of home affairs and the Leader of the
p a
u in the Lok Sabha.
Opposition
An
The Central Information Commission was established by the Central Government in 2005. The
Commission consists of a Chief Information Commissioner and not more than ten Information
Commissioners. They are appointed by the President on the recommendation of a committee
consisting of the Prime Minister as Chairperson, the Leader of Opposition in the Lok
Sabha and a Union Cabinet Minister nominated by the Prime Minister.

CEO of NITI Aayog is appointed by Prime Minister.

Hence, option (a) is correct.

20 Expenditure from which of the following accounts/funds is/are audited by the Comptroller and
Auditor General (CAG) ?
1. Consolidated Fund of India
2. Contingency Fund of India
3. Public Account of India

prelims.insightsonindia.com 13
© Insights Active Learning | All rights reserved - 131815. You may not reproduce, distribute or exploit the contents in any form without
written permission by copyright owner. Copyright infringers may face civil and criminal liability
Total Marks : 200
Online Prelims TEST - 9 (SUBJECT WISE)
( InsightsIAS Mock Test Series for UPSC Preliminary Exam 2020 ) Mark Scored : 76

Select the correct answer using the code given below.


A. 1 only
B. 1 and 2 only
C. 2 and 3 only
D. 1, 2 and 3

Your Answer : D
Correct Answer : D

Answer Justification :

All the statements are correct.

The duties and functions of the CAG as laid down by the Parliament and the Constitution are:
1. He audits the accounts related to all expenditure from the Consolidated Fund of India,
consolidated fund of each state and consolidated fund of each union territory having a Legislative
Assembly.
0 34
2. He audits all expenditure from the Contingency Fund of India and 4 1 Public Account of
the
6
54 of each state.
India as well as the contingency fund of each state and the public account
7
-8
m
o Rights (NCPCR), consider the
21 With reference to the National Commission for Protection ofcChild
i l .
ma
following statements:
1. It is a statutory body g
2
2. It works under the aegis of Ministry of Women2@and Child development
amincludes those up to the age of 14 years
3. As per the commission, definition of child
p
u
an
as above is/are correc
Which of the statements given
d
A. 1 and 2 only
s -
B. 2 only D a
C. 3 onlyam
p
nu2 and 3
D. 1,
A
Your Answer : D
Correct Answer : A

Answer Justification :

The National Commission for Protection of Child Rights (NCPCR) is an Indian governmental
commission, established by an Act of Parliament, the Commission for Protection of Child Rights Act
in December 2005, thus is a statutory body. Hence, statement 1 is coreect.

The commission works under the aegis of Ministry of Women and Child development, GoI.
The Commission began operation a year later in March 2007. Hence, statement 2 is correct.

The Commission considers that its Mandate is "to ensure that all Laws, Policies, Programmes, and
Administrative Mechanisms are in consonance with the Child Rights perspective as enshrined in the
Constitution of India and the UN Convention on the Rights of the Child."

prelims.insightsonindia.com 14
© Insights Active Learning | All rights reserved - 131815. You may not reproduce, distribute or exploit the contents in any form without
written permission by copyright owner. Copyright infringers may face civil and criminal liability
Total Marks : 200
Online Prelims TEST - 9 (SUBJECT WISE)
( InsightsIAS Mock Test Series for UPSC Preliminary Exam 2020 ) Mark Scored : 76

As defined by the commission, child includes those up to the age of 18 years. Hence,
statement 3 is incorrect.

22 With reference to the Advocate General of State, consider the following statements:
1. He is the highest law officer in the state.
2. It is appointed by the governor.
3. The term of office of the advocate general is fixed for 5 years by the Constitution.

Which of the statements given above is/are correct?


A. 1 and 2 only
B. 2 only
C. 1 and 3 only
D. 1, 2 and 3

Your Answer : C
4
Correct Answer : A
1 03
4 64
Answer Justification : 5
- 87
m advocate general for the states. He
The Constitution (Article 165) has provided for the office of the
o
il. c
is the highest law officer in the state. Thus he corresponds to the Attorney General of India.
Hence, statement 1 is correct. m a
2 @g
The advocate general is appointed by the
m 2 governor. He must be a person who is qualified to be
pa statement 2 is correct.
appointed a judge of a high court. Hence,
u
an
sadvocate
The term of office of the a general is not fixed by the Constitution. Further, the
d the procedure and grounds for his removal. He holds office during the
-
Constitution does not contain
s
Da This means that he may be removed by the governor at any time. Hence,
pleasure of the governor.
statement 3
p amis incorrect.
u
An
23 Consider the following statements:
1. VVPATs were first used in bye-election to the Noksen Assembly Constituency of Nagaland held in
2013.
2. Conducting exit polls and publishing results of exit polls would be prohibited during the election to
Lok Sabha and State Legislative Assemblies.

Select the correct answer from the codes given below:


A. 1 only
B. 2 only
C. Both 1 and 2
D. Neither 1 nor 2

Your Answer : C
Correct Answer : C

Answer Justification :

prelims.insightsonindia.com 15
© Insights Active Learning | All rights reserved - 131815. You may not reproduce, distribute or exploit the contents in any form without
written permission by copyright owner. Copyright infringers may face civil and criminal liability
Total Marks : 200
Online Prelims TEST - 9 (SUBJECT WISE)
( InsightsIAS Mock Test Series for UPSC Preliminary Exam 2020 ) Mark Scored : 76

VVPATs were first used in bye-election to the Noksen Assembly Constituency of Nagaland
held in 2013. Thereafter, VVPATs have been used in selected constituencies during every General
Election to State Legislative Assemblies. VVPATs were used in eight selected Parliamentary
Constituencies in the country in the 2014 Lok Sabha Election. Hence, statement 1 is correct.

According to a 2009 provision, conducting exit polls and publishing results of exit polls
would be prohibited during the election to Lok Sabha and State Legislative Assemblies.
Thus, no person shall conduct any exit poll and publish or publicise by means of the print or
electronic media or disseminate in any other manner, the result of any exit poll during the period
notified by the Election Commission in this regard. Hence, statement 2 is correct.

24 With reference to commissioners of Central Information Commission under RTI Act:


1. They should not be a Member of Parliament or Member of the Legislature of any State or Union
Territory.
2. They are appointed by the President on the recommendation of a committee consisting of the Prime
34 Minister
Minister as Chairperson, the Leader of Opposition in the Lok Sabha and a Union Cabinet
0
nominated by the Prime Minister. 1
3. Commissioners are appointed for duration of five years until they attain 4 64age of 62 years,
the
5
whichever is earlier. 87 -
Which of the statements given above is/are correct? .c om
i l
A. 1 and 2 only
g ma
2@
B. 2 only
C. 3 only
m 2
D. 1, 2 and 3 pa u
n
a sa
Your Answer : D d
Correct Answer : A s-
Da
am
Answer Justification :
nup
A
The Commission consists of a Chief Information Commissioner and not more than ten Information
Commissioners. They are appointed by the President on the recommendation of a
committee consisting of the Prime Minister as Chairperson, the Leader of Opposition in
the Lok Sabha and a Union Cabinet Minister nominated by the Prime Minister. Hence,
statement 2 is correct.

They should not be a Member of Parliament or Member of the Legislature of any State or
Union Territory. They should not hold any other office of profit or connected with any political
party or carrying on any business or pursuing any profession. Hence, statement 1 is correct.

The Chief Information Commissioner and an Information Commissioner hold office for a
term of 5 years or until they attain the age of 65 years, whichever is earlier. They are not
eligible for reappointment. Hence, statement 3 is incorrect.

25 With reference to Attorney General of India, consider the following statements:


1. He receives such remuneration as the president may determine.

prelims.insightsonindia.com 16
© Insights Active Learning | All rights reserved - 131815. You may not reproduce, distribute or exploit the contents in any form without
written permission by copyright owner. Copyright infringers may face civil and criminal liability
Total Marks : 200
Online Prelims TEST - 9 (SUBJECT WISE)
( InsightsIAS Mock Test Series for UPSC Preliminary Exam 2020 ) Mark Scored : 76

2. He must be a person who is qualified to be appointed a judge of the Supreme Court.

Which of the statements given above is/are correct?


A. 1 only
B. 2 only
C. Both 1 and 2
D. Neither 1 nor 2

Your Answer : C
Correct Answer : C

Answer Justification :

The Constitution (Article 76) has provided for the office of the Attorney General for India1. He is the
highest law officer in the country.

The Attorney General (AG) is appointed by the president. He must be a person


0 34 who is
qualified to be appointed a judge of the Supreme Court. In other words,
6 41he must be a citizen
of India and he must have been a judge of some high court for five years
7 54or an advocate of some
-8
high court for ten years or an eminent jurist, in the opinion of the president. Hence, statement 1
is correct. m
i l .co
m a
The remuneration of the AG is not fixed by the Constitution. He receives such remuneration as the
president may determine. Hence, statement @ g
2 is correct.
2
a m2
26 With reference to the Central Vigilance
n upCommission, which of the following statements is/are
correct?
a sa
1. It was established in 1964
s - dby an executive resolution of the Central government
Da
2. Presently, it is a statutory body.
a m
p
u correct answer using the codes given below:
Select the
A.A1nonly
B. 2 only
C. Both 1 and 2
D. Neither 1 nor 2

Your Answer : C
Correct Answer : C

Answer Justification :

The Central Vigilance Commission (CVC) is the main agency for preventing corruption in the
Central government. It was established in 1964 by an executive resolution of the Central
government. Its establishment was recommended by the Santhanam Committee on Prevention of
Corruption (1962–64). Hence, statement 1 is correct.

Thus, originally the CVC was neither a constitutional body nor a statutory body. Later, in
2003, the Parliament enacted a law conferring statutory status on the CVC. Hence,

prelims.insightsonindia.com 17
© Insights Active Learning | All rights reserved - 131815. You may not reproduce, distribute or exploit the contents in any form without
written permission by copyright owner. Copyright infringers may face civil and criminal liability
Total Marks : 200
Online Prelims TEST - 9 (SUBJECT WISE)
( InsightsIAS Mock Test Series for UPSC Preliminary Exam 2020 ) Mark Scored : 76

statement 2 is correct.

27 With reference to the State Information Commission, consider the following statements:
1. The Commission consists of a State Chief Information Commissioner and not more than five State
Information Commissioners.
2. Members of the commission are appointed by the President of India.
3. They are not eligible for reappointment.

Which of the statements given above is/are correct?


A. 1 only
B. 1 and 2 only
C. 3 only
D. 2 and 3 only

Your Answer : C
4
Correct Answer : C
1 03
4 64
Answer Justification : 5
- 87
m of not only the Central Information
The Right to Information Act of 2005 provides for the creation
o
.
Commission but also a State Information Commission atlthec state level.
i
m a
g Commissioner and not more than ten State
The Commission consists of a State Chief Information
@
2
Information Commissioners. Hence, statement2 1 is incorrect.
pam
u
anthe Leader of Opposition in the Legislative Assembly and a State
They are appointed by the Governor on the recommendation of a committee consisting of the
s
daby the Chief Minister. Hence, statement 2 is incorrect.
Chief Minister as Chairperson,
-
Cabinet Minister nominated
D as
m Information Commissioner and a State Information Commissioner hold office for a
The State Chief
pa or until they attain the age of 65 years, whichever is earlier. They are not eligible
term of 5uyears
An
for reappointment. Hence, statement 3 is correct.

28 With reference to the Indian Council of Agricultural Research (ICAR), consider the following
statements:
1. It is an autonomous organization under the Ministry of Agriculture and Farmers Welfare.
2. Prime minister serves as its president.
3. Recently, it has published an Integrated Mobile App called KISAAN (Krishi Integrated Solution for
Agri Apps Navigation) for Farmers of country.

Which of the statement(s) given above is/are correct?


A. 2 and 3 only
B. 1 and 3 only
C. 1 and 2 only
D. 1, 2 and 3

Your Answer : B

prelims.insightsonindia.com 18
© Insights Active Learning | All rights reserved - 131815. You may not reproduce, distribute or exploit the contents in any form without
written permission by copyright owner. Copyright infringers may face civil and criminal liability
Total Marks : 200
Online Prelims TEST - 9 (SUBJECT WISE)
( InsightsIAS Mock Test Series for UPSC Preliminary Exam 2020 ) Mark Scored : 76

Correct Answer : B

Answer Justification :

The Indian Council of Agricultural Research (ICAR) is an autonomous body responsible for
coordinating agricultural education and research in India. It reports to the Department of
Agricultural Research and Education, Ministry of Agriculture. Hence, statement 1 is
correct.

The Union Minister of Agriculture serves as its president. Hence, statement 2 is incorrect.

In 2019 ICAR has published an Integrated Mobile App called KISAAN (Krishi Integrated
Solution for Agri Apps Navigation) for Farmers of country. Mobile App has interface in 12
Indian Languages. Hence, statement 3 is correct.

29 Consider the following statements: 4


3Launching
1
1. Indian Space Research Organization (ISRO) set up the Thumba Equatorial Rocket 0
Station (TERLS) in Thiruvananthapuram for upper atmospheric research. 64
4
75
2. APSARA is the first nuclear reactor in Asia to have achieved criticality.
-8
Which of the statements given above is/are correct?
. c om
A. 1 only a il
B. 2 only
@ gm
C. Both 1 and 2 2 2
D. Neither 1 nor 2 am p
u
s an
Your Answer : a
Correct Answer : B
s -d
Da
am
Answer Justification :
n up
IndiaAdecided to go to space when Indian National Committee for Space Research (INCOSPAR) was
set up by the Government of India in 1962. With the visionary Dr Vikram Sarabhai at its helm,
INCOSPAR set up the Thumba Equatorial Rocket Launching Station (TERLS) in
Thiruvananthapuram for upper atmospheric research. Indian Space Research Organisation,
formed in 1969, superseded the erstwhile INCOSPAR. Hence, statement 1 is incorrect.

APSARA is the first nuclear reactor in Asia to have achieved criticality on August 4, 1956. It
is a pool type reactor of 1 MW power with a highly enriched uranium as fuel (4.5 kg) in the form of
plates. Light water was used as both moderator and coolant. Hence, statement 2 is correct.

http://www.barc.gov.in/about/index.html#

https://www.isro.gov.in/about-isro

30 Consider the following statements:


1. Telecom Regulatory Authority of India is a non-statutory executive body established by an executive

prelims.insightsonindia.com 19
© Insights Active Learning | All rights reserved - 131815. You may not reproduce, distribute or exploit the contents in any form without
written permission by copyright owner. Copyright infringers may face civil and criminal liability
Total Marks : 200
Online Prelims TEST - 9 (SUBJECT WISE)
( InsightsIAS Mock Test Series for UPSC Preliminary Exam 2020 ) Mark Scored : 76

resolution.
2. The National Commission for Safai Karamcharis (NCSK) was constituted as a statutory body, but
presently it is working as a non-statutory body.
3. The National Commission for Denotified, Nomadic and Semi-Nomadic Tribes is a national
commission set under the Ministry of Tribal Affairs.

Which of the statements given above is/are correct?


A. 2 only
B. 1 and 3
C. 3 only
D. 1, 2 and 3

Your Answer : A
Correct Answer : A

Answer Justification :
0 34
The Telecom Regulatory Authority of India (TRAI) was, thus, established
6 41 with effect from
20th February 1997 by an Act of Parliament, called the Telecom 5 4
Regulatory Authority of India
7
Act, 1997, to regulate telecom services, including fixation/revision - 8 of tariffs for telecom services
which were earlier vested in the Central Government. Hence,m
. c o statement 1 is incorrect.
a il
gm
The National Commission for Safai Karamcharis (NCSK) was constituted on 12th August,
1994 as a statutory body by an Act of@ Parliament viz. ‘National Commission for Safai
2
Karamcharis Act, 1993’, for a period of three
2years i.e. up to 31st March, 1997.
m
u pa
With the lapsing of the “The n National Commission for Safai Karamcharis Act, 1993” w.e.f.
29.2.2004, the Commission a sisa acting as a Non-Statutory body of the Ministry of Social Justice
and Empowerment whoses - dtenure is extended from time to time through Government Resolutions.
Da
am 2 is correct.
Hence, statement
p
The A
nu
National Commission for Denotified, Nomadic and Semi-Nomadic Tribes is a national
commission set under the Ministry of Social Justice and Empowerment to study various
developmental aspects of denotified and nomadic or semi-nomadic tribes in India. Hence,
statement 3 is incorrect.

https://main.trai.gov.in/about-us/history

https://ncsk.nic.in/about-us/about-ncsk

31 With reference to National Commission for Women, consider the following statements:
1. It is a statutory body.
2. It take suo-moto notice of matters relating to deprivation of women's rights.
3. Among the five members of the commission at least one Member each shall be from amongst
persons belonging to the Scheduled Castes and Scheduled Tribes respectively.

Which of the statements given above is/are correct?

prelims.insightsonindia.com 20
© Insights Active Learning | All rights reserved - 131815. You may not reproduce, distribute or exploit the contents in any form without
written permission by copyright owner. Copyright infringers may face civil and criminal liability
Total Marks : 200
Online Prelims TEST - 9 (SUBJECT WISE)
( InsightsIAS Mock Test Series for UPSC Preliminary Exam 2020 ) Mark Scored : 76

A. 1 only
B. 2 and 3
C. 1 and 3
D. 1, 2 and 3

Your Answer : D
Correct Answer : D

Answer Justification :

The National Commission for Women was set up as statutory body in January 1992 under
the National Commission for Women Act, 1990 ( Act No. 20 of 1990 of Govt.of India ) to :

review the Constitutional and Legal safeguards for women ;

4
recommend remedial legislative measures ; 1 03
4 64
5
facilitate redressal of grievances and - 87
.c om
i l
g ma
advise the Government on all policy matters affecting women.

2 2@
m
pa
Hence, statement 1 is correct.
n u
sacomplaints and take suo moto notice of matters relating to:-
The Commission shall look into
a
s -d
a
Dof
1. deprivation women's rights. Hence, statement 2 is correct.
a m
n up
2.Anon-implementation of laws enacted to provide protection to women and also to achieve the
objective of equality and development,

3. non-compliance of policy decisions,guidelines or instructions aimed at mitigating hardships


and ensuring welfare and providing relief to women, and take up the issues arising out of
such matters with appropriate authorities.

http://ncw.nic.in/commission/about-us

http://ncw.nic.in/commission/about-us/constitution

http://ncw.nic.in/commission/about-us/mandate

The Commission shall consist of :-

prelims.insightsonindia.com 21
© Insights Active Learning | All rights reserved - 131815. You may not reproduce, distribute or exploit the contents in any form without
written permission by copyright owner. Copyright infringers may face civil and criminal liability
Total Marks : 200
Online Prelims TEST - 9 (SUBJECT WISE)
( InsightsIAS Mock Test Series for UPSC Preliminary Exam 2020 ) Mark Scored : 76

1. A Chairperson, committed to the cause of women, to be nominated by the Central


Government.

2. five Members to be nominated by the Central Government from amongst persons of ability,
integrity and standing who have had experience in law or legislation, trade unionism,
management of an industry potential of women, women's voluntary organisations ( including
women activist ), administration, economic development, health, education or social welfare;
Provided that at least one Member each shall be from amongst persons belonging to
the Scheduled Castes and Scheduled Tribes respectively. Hence, statement 3 is
correct.

32 Consider the following statements:


1. India Meteorological Department (IMD) works under the Ministry of Environment, Forest and
Climate Change (MoEFCC)
4 adverse
2. It is the duty of the Competition Commission of India (CCI) to eliminate practices having
3
10consumers and
effect on competition, promote and sustain competition, protect the interests of
4
ensure freedom of trade in the markets of India 4 6
7 5
Which of the statements given above is/are correct? -8
A. 1 only
. c om
B. 2 only a il
C. Both 1 and 2
@ gm
D. Neither 1 nor 2 22 m
u pa
Your Answer : B
a n
s
Correct Answer : B
da
a s-
D :
Answer Justification
a m
u
The India pMeteorological Department (IMD) is an agency of the Ministry of Earth
A n
Sciences of the Government of India. It is the principal agency responsible for
meteorological observations, weather forecasting and seismology. Hence, statement 1 is
incorrect.

IMD is also one of the six Regional Specialised Meteorological Centres of the World Meteorological
Organization. It has the responsibility for forecasting, naming and distribution of warnings
for tropical cyclones in the Northern Indian Ocean region, including the Malacca Straits, the Bay of
Bengal, the Arabian Sea and the Persian Gulf.

Competition Commission of India is a statutory body of the Government of India responsible for
enforcing The Competition Act, 2002 throughout India and to prevent activities that have an
appreciable adverse effect on competition in India.

It is the duty of the Commission to eliminate practices having adverse effect on competition,
promote and sustain competition, protect the interests of consumers and ensure freedom
of trade in the markets of India. The Commission is also required to give an opinion on
competition issues on a reference received from a statutory authority established under any law and

prelims.insightsonindia.com 22
© Insights Active Learning | All rights reserved - 131815. You may not reproduce, distribute or exploit the contents in any form without
written permission by copyright owner. Copyright infringers may face civil and criminal liability
Total Marks : 200
Online Prelims TEST - 9 (SUBJECT WISE)
( InsightsIAS Mock Test Series for UPSC Preliminary Exam 2020 ) Mark Scored : 76

to undertake competition advocacy, create public awareness and impart training on competition
issues. Hence, statement 2 is correct.

http://imd.gov.in/pages/about_organisation.php

https://www.cci.gov.in/about-cci

33 Consider the following statements:


1. The Central Electricity Authority of India (CEA) is a statutory organization .
2. Council of Scientific and Industrial Research operates as an autonomous body through the Societies
Registration Act, 1860

Which of the statements given above is/are correct?


A. 1 only
B. 2 only
C. Both 1 and 2 4
D. Neither 1 nor 2 1 03
4 64
5
Your Answer : C
- 87
Correct Answer : C
.com
Answer Justification : a i l
@ gm
2 2
Central Electricity Authority (CEA) is an organization originally constituted under Section 3(1)
of the repealed Electricity (Supply)aAct,m 1948, since substituted by Section 70 of the
u p
aofnCEA are delineated under Section 73 of the Electricity Act, 2003.
Electricity Act, 2003. It was established as a part-time body in 1951 and made a full-time body in
s
da
1975. The functions and duties
Hence, statement 1 is-correct.
D as
inp
of India u
amScientific and Industrial Research (CSIR) was established by the Government
The Council of
September of 1942 as an autonomous body that has emerged as the largest research
and A n
development organisation in India. Although it is mainly funded by the Ministry of Science and
Technology, it operates as an autonomous body through the Societies Registration Act, 1860.
Hence, statement 2 is correct.

http://cea.nic.in/aboutus.html

34 Consider the following statements regarding National Foundation for Communal Harmony
1. It is an autonomous body under the administrative control of the Union Home Ministry.
2. It provides financial assistance to the child victims of societal violence for their care, education and
training.

Which of the statements given above is/are correct?


A. 1 only
B. 2 only
C. Both 1 and 2
D. Neither 1 nor 2

prelims.insightsonindia.com 23
© Insights Active Learning | All rights reserved - 131815. You may not reproduce, distribute or exploit the contents in any form without
written permission by copyright owner. Copyright infringers may face civil and criminal liability
Total Marks : 200
Online Prelims TEST - 9 (SUBJECT WISE)
( InsightsIAS Mock Test Series for UPSC Preliminary Exam 2020 ) Mark Scored : 76

Your Answer : C
Correct Answer : C

Answer Justification :

The National Foundation for Communal Harmony (NFCH) was set up in 1992. It is an
autonomous body under the administrative control of the Union Home Ministry. It
promotes communal harmony, fraternity and national integration. Hence Statement 1 is
correct.

The activities undertaken by the NFCH are mentioned below:

1. To provide financial assistance to the child victims of societal violence for their care,
education and training, aimed at their effective rehabilitation Hence Statement 2 is
correct.

2. To promote communal harmony and national integration by organising variety of4activities either
3
10
independently or in association with educational institutions, NGOs & other organisations
4 64
5 conducting studies
3. To conduct studies and grant scholarships to institutions / scholars7for
- 8
m
i l
35 Which of the following is/are the principles of Indian Foreign.coPolicy?
1. Promotion of World Peace
g ma
2. Anti-Colonialism
2 2@
3. Non-Alignment m
4. Links with Commonwealth u pa
n
a sa
Select the correct answerd
A. 1 and 2 only as
- using the code given below
D
B. 1, 2 and 3 only
C. 2, 3 p
and
m
a 4 only
u
n 2, 3 and 4
D.A1,

Your Answer : D
Correct Answer : D

Answer Justification :

Principles of Indian Foreign Policy

• Promotion of World Peace

• Anti-Colonialism

• Anti-Racialism

• Non-Alignment

• Panchsheel

prelims.insightsonindia.com 24
© Insights Active Learning | All rights reserved - 131815. You may not reproduce, distribute or exploit the contents in any form without
written permission by copyright owner. Copyright infringers may face civil and criminal liability
Total Marks : 200
Online Prelims TEST - 9 (SUBJECT WISE)
( InsightsIAS Mock Test Series for UPSC Preliminary Exam 2020 ) Mark Scored : 76

• Afro-Asian Bias

• Links with Commonwealth

• Support to the UNO

Hence, option (d) is correct.

36 Consider the following statements


1. India’s Look East Policy was first initiated by I.K. Gujral
2. No South Asian country should allow its territory to be used against the interest of another country
of the region is one of the features of Gujral Doctrine

Which of the statements given above is/are correct?


A. 1 only
B. 2 only 4
C. Both 1 and 2 1 03
D. Neither 1 nor 2
4 64
5
Your Answer : - 87
Correct Answer : B
.com
a i l
Answer Justification : g m
2 2@
p am
India’s ‘Look East’ Policy was first initiated in 1992 by the then Prime Minister P.V.
Narasimha Rao. Hence Statement u 1 is incorrect.
n
a sa
Since then, the policy hasd
s - been one of the cornerstones of India’s foreign policy. The policy is a
a vision of the world and India’s place in the evolving global economy. It lays
strategic shift in India’s
D
emphasis on improving cooperation with India’s neighbouring south-east and East Asian countries.
p am
Gujral nu
A doctrine is a five-point roadmap to guide the conduct of India’s foreign relations with its
immediate neighbours. These five principles are as follows:

1. With the neighbors like Bangladesh, Bhutan, Maldives, Nepal and Sri Lanka, India should not ask
for reciprocity, but give to them what it can in good faith.

2. No South Asian country should allow its territory to be used against the interest of
another country of the region. Hence Statement 2 is correct.

3. No country should interfere in the internal affairs of another country.

4. All South Asian countries should respect each other’s territorial integrity and sovereignty.

5. All South Asian countries should settle all their disputes through peaceful bilateral negotiations.

prelims.insightsonindia.com 25
© Insights Active Learning | All rights reserved - 131815. You may not reproduce, distribute or exploit the contents in any form without
written permission by copyright owner. Copyright infringers may face civil and criminal liability
Total Marks : 200
Online Prelims TEST - 9 (SUBJECT WISE)
( InsightsIAS Mock Test Series for UPSC Preliminary Exam 2020 ) Mark Scored : 76

37 Consider the following statements regarding Nuclear Doctrine of India


1. Nuclear retaliatory attacks can only be authorised by the civilian political leadership
2. Non-use of nuclear weapons against non-nuclear weapon states
3. Building and maintaining a credible minimum deterrent.

Which of the statements given above is/are correct?


A. 1 and 2 only
B. 2 and 3 only
C. 3 only
D. 1, 2 and 3

Your Answer : D
Correct Answer : D

Answer Justification :
4
All the statements given above are correct.
1 03
4 64
Nuclear Doctrine of India 5
- 87
mof this doctrine are as follows:
India adopted its nuclear doctrine in 2003. The salient features
i l .co
ma
1. Building and maintaining a credible minimum deterrent.
g
2
2. A posture of “No First Use” – nuclear weapons2@ will only be used in retaliation against a nuclear
m anywhere.
attack on Indian territory or on Indianaforces
u p
n
a sastrike will be massive and designed to inflict unacceptable damage.
3. Nuclear retaliation to a first

s -d
a attacks can only be authorised by the civilian political leadership through the
4. Nuclear retaliatory
D
am Authority.
Nuclear Command
p
u
An of nuclear weapons against non-nuclear weapon states.
5. Non-use

6. However, in the event of a major attack against India, or Indian forces anywhere, by biological or
chemical weapons, India will retain the option of retaliating with nuclear weapons.

7. A continuance of strict controls on export of nuclear and missile related materials and
technologies, participation in the Fissile Material Cutoff Treaty negotiations, and continued
observance of the moratorium on nuclear tests.

8. Continued commitment to the goal of a nuclear-weapon-free world, through global, verifiable and
non-discriminatory nuclear disarmament.

38 Cabinet committee on Security consists of which of the following members?


1. Prime Minister
2. Minister of Home Affairs
3. Minister of External Affairs
4. Ministry of Road Transport and Highways

prelims.insightsonindia.com 26
© Insights Active Learning | All rights reserved - 131815. You may not reproduce, distribute or exploit the contents in any form without
written permission by copyright owner. Copyright infringers may face civil and criminal liability
Total Marks : 200
Online Prelims TEST - 9 (SUBJECT WISE)
( InsightsIAS Mock Test Series for UPSC Preliminary Exam 2020 ) Mark Scored : 76

Select the correct answer using the code given below


A. 1 and 3 only
B. 1, 2 and 3 only
C. 1 and 2 only
D. 1, 2, 3 and 4

Your Answer : B
Correct Answer : B

Answer Justification :

The Cabinet Committee on Security (CCS) of the Central Government of India discusses, debates
and is the final decision-making body on senior appointments in the national security apparatus
,defence policy and expenditure, and generally all matters of India's national security. The CCS is
chaired by the Prime Minister of India. The CCS consists of the following members:

4
Prime Minister
1 03
Minister of Home Affairs 4 64
5
- 87
Minister of Defence
.com
Minister of External Affairs a i l
g m
Minister of Finance and Corporate Affairs.
2 2@
am
Hence, option (b) is correct. up
s an
da
s - have a bearing on health?
39 Which of these laws in India
a
D Psychotropic Substances Act, 1985
1. Narcotic Drugs and
m
a Food Adulteration Act, 1954
2. The Prevention of
u p
n Waste (Management and Handling) Rules, 1998
3. Bio-Medical
A
4. Consumer Protection Act, 1986

Select the correct answer using the codes below.


A. 1 and 2 only
B. 3 and 4 only
C. 1, 2 and 3 only
D. 1, 2, 3 and 4

Your Answer : D
Correct Answer : D

Answer Justification :

All the above laws have bearing on Health

The Act prohibits a person to produce/manufacture/cultivate, possess, sell, purchase, transport,

prelims.insightsonindia.com 27
© Insights Active Learning | All rights reserved - 131815. You may not reproduce, distribute or exploit the contents in any form without
written permission by copyright owner. Copyright infringers may face civil and criminal liability
Total Marks : 200
Online Prelims TEST - 9 (SUBJECT WISE)
( InsightsIAS Mock Test Series for UPSC Preliminary Exam 2020 ) Mark Scored : 76

store, and/or consume any narcotic drug or psychotropic substance which has a direct bearing on
personal and social health.

Food adulteration, such as mixing of ammonia in milk, significantly risks public health and is thus
regulated by the Act.

Bio-medical wastes emitted by hospitals and other such institutions are regulated by this act, which
could become a local menace by leaching into groundwater, spread microorganisms etc.

Suppose a wrong medicine was prescribed by a hospital to patient, consuming which causes
immense damage to the patient. The patient, as a consumer, can raise this issue in the consumer
court and get adequate compensation.

40 How is the World Bank Group (WBG) President selected?

A. Nominated by the largest shareholder of WBG subject to confirmation by the 4Board of


0 3
41
Governors
B. Elected on a majority basis by the shareholder member states of the6
WBG
C. Elected by the United Nations General Assembly (UNGA) on a 7 54
two-thirds majority basis
- 8
D. Appointed by the UN Secretary General subject to approval by the Board of Governors
o m
Your Answer : a il.c
m
@g
Correct Answer : A
2 2
m
pa
Answer Justification :
n u
a sa
Traditionally, the Bank President has always been a U.S. citizen nominated by the
President of the United - dStates, the largest shareholder in the bank. The nominee is subject
s
a Board of Governors, to serve for a five-year, renewable term. The president is
to confirmation by the
D
responsible formchairing the meetings of the Boards of Directors and for overall management of the
aGroup.
u
World Bank p Hence, option (a) is correct.
An
41 Consider the following statements regarding Financial Action Task Force (FATF)
1. It was set up in 1989 by International Monetary Fund with headquarters in Paris.
2. FATF acts as an international watchdog on issues of money-laundering and financing of terrorism.
3. The Gulf Cooperation Council (GCC) and the European Commission (EC) are members of it.

Which of the statements given above is/are correct?


A. 1 and 3 only
B. 2 and 3 only
C. 1 and 2 only
D. 1, 2 and 3

Your Answer : B
Correct Answer : B

Answer Justification :

prelims.insightsonindia.com 28
© Insights Active Learning | All rights reserved - 131815. You may not reproduce, distribute or exploit the contents in any form without
written permission by copyright owner. Copyright infringers may face civil and criminal liability
Total Marks : 200
Online Prelims TEST - 9 (SUBJECT WISE)
( InsightsIAS Mock Test Series for UPSC Preliminary Exam 2020 ) Mark Scored : 76

Financial Action Task Force was set up in 1989 by the G7 countries with headquarters in
Paris. Hence Statement 1 is incorrect.

FATF acts as an “international watchdog‟ on issues of money-laundering and financing of


terrorism. It has 37 members, which include all 5 permanent members of the Security Council and
other countries with economic influence. Hence Statement 2 is correct.

Two regional organisations, the Gulf Cooperation Council (GCC) and the European
Commission (EC) are also its members. Hence Statement 3 is correct.

https://www.fatf-gafi.org/about/

42 Consider the following statements regarding BRICS


1. South Africa joined BRICS in 2010
2. Last summit of BRICS was held at China.
3. BRICS established The Asian Infrastructure Investment Bank 4
1 03
Which of the statements given above is/are correct?
4 64
5
A. 1 only
B. 2 only - 87
C. 1 and 3 only
.com
D. 1, 2 and 3 a i l
g m
Your Answer : A 2 2@
Correct Answer : A am
up
s an
da
Answer Justification :
-
BRICS is the acronym
D ascoined for an association of five major emerging national economies: Brazil,
Russia, India, m
China and South Africa. Originally the first four were grouped as "BRIC" (or "the
pa the induction of South Africa in 2010. Hence Statement 1 is correct.
BRICs"),ubefore
An
Last summit of BRICS was held at South Africa. Hence Statement 2 is incorrect.

The proposal for the creation of an "Asian Infrastructure Investment Bank" was first made by
the Vice Chairman of the China Center for International Economic Exchanges, a Chinese thinktank,
at the Bo'ao Forum in April 2009. The initiative was officially launched by Chinese President
Xi Jinping on a state visit to Indonesia in October 2013. Hence Statement 3 is incorrect.

43 Consider the following statements


1. The Nuclear Suppliers Group (NSG) was set up in 1974 as a reaction to India’s nuclear tests to stop
what it called the misuse of nuclear material meant for peaceful purposes.
2. The NSG works under the principle of unanimity and even one country’s vote against prospective
member will scuttle its bid.
3. India, Pakistan, Israel, South Sudan and North Korea are among the four UN member states which
have not signed the NPT.

prelims.insightsonindia.com 29
© Insights Active Learning | All rights reserved - 131815. You may not reproduce, distribute or exploit the contents in any form without
written permission by copyright owner. Copyright infringers may face civil and criminal liability
Total Marks : 200
Online Prelims TEST - 9 (SUBJECT WISE)
( InsightsIAS Mock Test Series for UPSC Preliminary Exam 2020 ) Mark Scored : 76

Which of the following statements given above is/are correct?


A. 1 only
B. 2 and 3 only
C. 1 and 2 only
D. 1, 2 and 3

Your Answer : D
Correct Answer : C

Answer Justification :

The NSG was founded in response to the Indian nuclear test in May 1974 and first met in
November 1975. The test demonstrated that certain non-weapons specific nuclear technology could
be readily turned to weapons development. Hence Statement 1 is correct.

The NSG looks after critical issues relating to nuclear sector and its members are allowed to trade
in and export nuclear technology. The group works under the principle of unanimity
0 34 and even
one country's vote against India will scuttle its bid. Hence Statement421is correct.
4 6
As of August 2016, 191 states have adhered to the treaty, though 75
8 North Korea, which
acceded in 1985 but never came into compliance, announced m
-
its withdrawal from the NPT in 2003,
c o
. obligations. Hence Statement 3 is
ail
following detonation of nuclear devices in violation of core
incorrect.
g m
2
https://en.wikipedia.org/wiki/Nuclear_Suppliers_Group 2@
m
u pa
n
sa regarding Group of 7 (G7) countries
44 Consider the following statements
a
1. G7 is a group consisting-ofdCanada, France, Germany, Italy, Japan, the United Kingdom, and the
s
United States.
Da
m population amongst G7 countries
2. Canada has least
a
p
u represent more than 40% of the net global wealth.
3. G7 countries
An
Which of the statements given above is/are correct?
A. 1 only
B. 1 and 3 only
C. 2 and 3 only
D. 1, 2 and 3

Your Answer : D
Correct Answer : D

Answer Justification :

All the statements given above are correct.

The Group of 7 (G7) is a group consisting of Canada, France, Germany, Italy, Japan, the United
Kingdom, and the United States. The European Union is also represented within the G7.

prelims.insightsonindia.com 30
© Insights Active Learning | All rights reserved - 131815. You may not reproduce, distribute or exploit the contents in any form without
written permission by copyright owner. Copyright infringers may face civil and criminal liability
Total Marks : 200
Online Prelims TEST - 9 (SUBJECT WISE)
( InsightsIAS Mock Test Series for UPSC Preliminary Exam 2020 ) Mark Scored : 76

As of 2018, the seven countries involved represent 58% of the global net wealth ($317
trillion) and more than 46% of the global gross domestic product (GDP) based on nominal values,
and more than 32% of the global GDP based on purchasing power parity.

Canada has least population amongst G7 countries (35,467,000).

https://en.wikipedia.org/wiki/Group_of_Seven

45 The Global Infrastructure Facility, sometimes seen in the news, is an initiative by

A. The International Monetary Fund


B. The World Bank
C. The World Economic Forum
D. The United Nations Conference on Trade and Development

Your Answer : 4
Correct Answer : B 1 03
4 64
5
Answer Justification :
- 87
m
The Global Infrastructure Facility (GIF) is a partnership
i l . co among governments, multilateral
development banks, private sector investors, andafinanciers. It is designed to provide a new
m
way to collaborate on preparing, structuring, andgimplementing complex projects that no single
2 @
m2
institution could handle on its own.
a
The World Bank (WB) has launched
n up the Global Infrastructure Facility.
a sa
-d
https://www.globalinfrafacility.org/what-is-the-gif
s
Da
amIAEA Low Enriched Uranium (LEU) bank had established in
46 The world's first
p
A nu
A. Japan
B. United States
C. France
D. Kazakhstan

Your Answer : D
Correct Answer : D

Answer Justification :

The IAEA LEU Bank will be a reserve of LEU owned and controlled by the IAEA, and a
mechanism of last resort for Member States in case the supply of LEU to a nuclear power plant is
disrupted due to exceptional circumstances and the Member State is unable to secure LEU from the
commercial market or by any other means.

World’s first IAEA was established in Kazakhstan.

prelims.insightsonindia.com 31
© Insights Active Learning | All rights reserved - 131815. You may not reproduce, distribute or exploit the contents in any form without
written permission by copyright owner. Copyright infringers may face civil and criminal liability
Total Marks : 200
Online Prelims TEST - 9 (SUBJECT WISE)
( InsightsIAS Mock Test Series for UPSC Preliminary Exam 2020 ) Mark Scored : 76

https://www.iaea.org/topics/iaea-low-enriched-uranium-bank

47 Consider the following statements


1. Muslims, Sikhs, Christians, Buddhists, Jain and Zorastrians (Parsis) have been notified as minority
communities under of the National Commission for Minorities Act, 1992.
2. Prime Minister's Twenty Point Programme for minorities is a programme launched by Indian
government in 2006 for welfare of religious minorities.
3. National Minorities Development & Finance Corporation (NMDFC) aims to promote the economic &
developmental activities for the benefit of all sections of notified Minorities

Which of the statements given above is/are correct?


A. 2 only
B. 1 only
C. 1 and 3 only
D. 1, 2 and 3
4
1 03
Your Answer : D
4 64
Correct Answer : B 5
- 87
Answer Justification :
.c om
i l
ma (Parsis) have been notified as minority
Muslims, Sikhs, Christians, Buddhists, Jain and Zorastrians
g
communities under of the National Commission
2 @for Minorities Act, 1992. Hence Statement 1 is
correct.
a m2
n up for minorities is a programme launched by Indian
Prime Minister's 15 point programme
a
government in 2006 for welfaresaof religious minorities. Hence Statement 2 is incorrect.
s -d
DaDevelopment & Finance Corporation (NMDFC) aims to promote the economic &
National Minorities
amactivities for the benefit of backward sections of notified minorities. Hence
developmental
p
u 3 is incorrect.
Statement
An
48 Consider the following statements
1. Organisation of Islamic Cooperation (OIC) is a regional organization within European Union.
2. Arab League is the collective voice of the Muslim world and works to safeguard and protect the
interests of the Muslim world in the spirit of promoting international peace and harmony

Which of the statements given above is/are correct?


A. 1 only
B. 2 only
C. Both 1 and 2
D. Neither 1 nor 2

Your Answer : D
Correct Answer : D

Answer Justification :

prelims.insightsonindia.com 32
© Insights Active Learning | All rights reserved - 131815. You may not reproduce, distribute or exploit the contents in any form without
written permission by copyright owner. Copyright infringers may face civil and criminal liability
Total Marks : 200
Online Prelims TEST - 9 (SUBJECT WISE)
( InsightsIAS Mock Test Series for UPSC Preliminary Exam 2020 ) Mark Scored : 76

Organisation of Islamic Cooperation (OIC) is a regional organization but not within European
Union. Hence Statement 1 is incorrect.

Organisation of Islamic Cooperation (OIC) is the collective voice of the Muslim world and works to
safeguard and protect the interests of the Muslim world in the spirit of promoting international
peace and harmony. Hence Statement 2 is incorrect.

https://www.oic-oci.org/page/?p_id=52&p_ref=26&lan=en

https://en.wikipedia.org/wiki/Organisation_of_Islamic_Cooperation

49 Consider the following statements


1. The Sangeet Natak Akademi is one of the first National Academy of the arts set-up by the Republic
of India.
2. The Sangeet Natak Akademi is presently an autonomous body of the Ministry of Culture,
4 schemes and
Government of India and is fully funded by the Government for implementation of its
3
0
41
programmes.
4 6
Which of the statement given above is/are correct? 7 5
A. 1 only -8
B. 2 only
. c om
C. Both 1 and 2 a il
D. Neither 1 nor 2 gm @
2 2
m
pa
Your Answer : C
Correct Answer : C n u
s a
Answer Justification :- da
D as
p am
Both the statements are correct.

The A nu Natak Akademi - India's national academy for music, dance and drama - is the first
Sangeet
National Academy of the arts set-up by the Republic of India. It was created by a resolution of
the (then) Ministry of Education, Government of India, dated 31 May 1952 notified in the Gazette of
India of June 1952.

The Akademi became functional the following year, with the appointment of its first Chairman, Dr
P.V. Rajamannar, and the formation of its all-India council of representatives, the General Council.

The Sangeet Natak Akademi is presently an Autonomous Body of the Ministry of Culture,
Government of India and is fully funded by the Government for implementation of its
schemes and programmes.

https://www.sangeetnatak.gov.in/sna/introduction.php

50 Section 8 of the Representation of the People (RP) Act, 1951 that is frequently in news concerns

prelims.insightsonindia.com 33
© Insights Active Learning | All rights reserved - 131815. You may not reproduce, distribute or exploit the contents in any form without
written permission by copyright owner. Copyright infringers may face civil and criminal liability
Total Marks : 200
Online Prelims TEST - 9 (SUBJECT WISE)
( InsightsIAS Mock Test Series for UPSC Preliminary Exam 2020 ) Mark Scored : 76

A. Paid advertisements for election campaign.


B. Division of seats in the Upper house.
C. Horse trading in council elections.
D. Disqualification on conviction for long sentences.

Your Answer : D
Correct Answer : D

Answer Justification :

Section 8 disqualifies a person convicted with a sentence of two years or more from
contesting elections. But those under trial continued to be eligible to contest elections. The Lily
Thomas case (2013), however, ended this unfair advantage.

Currently, under the Representation of Peoples (RP) Act, lawmakers cannot contest elections only
after their conviction in a criminal case.
4
1 03
51 Consider the following statements
4 64
7 5difficult
1. Swadhar is a central sector scheme for integrated services to women8 in
- family
circumstances like
destitute widows, women prisoners released from jail and without support, women survivors
of natural disasters etc. . c om
il rehabilitation facilities for women
2. Sakhi is a one-stop centre for providing medical, legalaand
subjected to any kind of violence. gm @
2 2
m
a not correct?
Which of the statements given above is/are
u p
A. 1 only n
B. 2 only a sa
C. Both 1 and 2 s -
d
Da2
D. Neither 1 nor

p am
nu : C
Your Answer
A
Correct Answer : D

Answer Justification :

Both the statements are correct.

The scheme envisions a supportive institutional framework for women victims of difficult
circumstances so that they could lead their life with dignity and conviction. It envisages that
shelter, food, clothing, and health as well as economic and social security are assured for such
women. It also envisions that the special needs of these women are properly taken care of and
under no circumstances they should be left unattended or abandoned which could lead to their
exploitation and desolation

http://vikaspedia.in/social-welfare/women-and-child-development/women-development-1/swadhar

http://vikaspedia.in/social-welfare/women-and-child-development/women-development-1/poorna-sha
kti-kendras

prelims.insightsonindia.com 34
© Insights Active Learning | All rights reserved - 131815. You may not reproduce, distribute or exploit the contents in any form without
written permission by copyright owner. Copyright infringers may face civil and criminal liability
Total Marks : 200
Online Prelims TEST - 9 (SUBJECT WISE)
( InsightsIAS Mock Test Series for UPSC Preliminary Exam 2020 ) Mark Scored : 76

52 Consider the following statements regarding Organisation for Economic Co-operation and
Development (OECD)
1. Russia is a founding member of OECD.
2. It is an intergovernmental economic organisation, founded to stimulate economic progress and
world trade.
3. Most OECD members are high-income economies with a very high Human Development Index (HDI)
and are regarded as developed countries.

Which of the statements given above is/are correct?


A. 2 only
B. 2 and 3 only
C. 3 only
D. 1 and 2 only

Your Answer : B
Correct Answer : B 4
1 03
Answer Justification :
4 64
5
-
Russia is not a member of OECD itself. Hence Statement 1 is incorrect. 87
.c om
i l
The Organisation for Economic Co-operation and Development
a
is an intergovernmental
mfounded in 1961 to stimulate economic
economic organisation with 36 member countries,
g
progress and world trade. Hence Statement @
2 is correct.
2 2
m
pa economies with a very high Human Development
Most OECD members are high-income
u
anas developed countries. Hence Statement 3 is correct.
Index (HDI) and are regarded
s
a
s
https://www.oecd.org/about/ -d
Da
am
https://en.wikipedia.org/wiki/OECD
nup
A
53 Consider the following statements regarding Raisina Dialogue
1. The Raisina Dialogue is India’s flagship annual geopolitical and geostrategic conference.
2. It is jointly organised by the Ministry of External Affairs (MEA) and Observer Research Foundation
(ORF).
3. The theme of the 2019 conference is Managing Disruptive Transitions: Ideas, Institutions and
Idioms

Which of the statements given above is/are correct?


A. 2 only
B. 1 and 3 only
C. 3 only
D. 1 and 2 only

Your Answer : D
Correct Answer : D

prelims.insightsonindia.com 35
© Insights Active Learning | All rights reserved - 131815. You may not reproduce, distribute or exploit the contents in any form without
written permission by copyright owner. Copyright infringers may face civil and criminal liability
Total Marks : 200
Online Prelims TEST - 9 (SUBJECT WISE)
( InsightsIAS Mock Test Series for UPSC Preliminary Exam 2020 ) Mark Scored : 76

Answer Justification :

The Raisina Dialogue is India’s flagship annual geopolitical and geostrategic conference.
Statement 1 is correct.

Ministry of External Affairs in partnership with Observer Research Foundation is organizing the 4th
edition of the Raisina Dialogue in New Delhi from 08-10 January 2019. Statement 2 is correct.

2019 conference theme is "A World Reorder: New Geometries; Fluid Partnerships; Uncertain
Outcomes”. The discussions will seek to address issues arising from ongoing global transitions and
changes to the world order, triggered by unique leaders, innovative partnerships and new
technologies. Statement 3 is incorrect.

54 Member countries of Indian Ocean Rim Association (IORA) are

A. India, Egypt, Sri Lanka, Bangladesh and New Zealand 4


B. India, Malaysia, Saudi Arabia, South Africa and Iraq 1 03
C. India, Australia, Iran, Indonesia and Thailand 4 64
5
D. India, China, Singapore, Mauritius and Madagascar
- 87
Your Answer : D .com
a i l
Correct Answer : C
g m
2@
m2
Answer Justification :
pa
n
India, Australia, Iran IR, Indonesia uThailand, Malaysia, South Africa, Mozambique, Kenya, Sri
s a
da
Lanka, Tanzania, Bangladesh, Singapore, Mauritius, Madagascar, UAE, Yemen, Seychelles, Somalia,
Comoros and Oman are-members of IORA.
s
Da
am
http://pib.nic.in/newsite/PrintRelease.aspx?relid=183973
p
u
An
55 Consider the following statements regarding International North South Transport Corridor (INSTC)
1. Multi modal transport corridor signed in 2000 in St.Petersburg.
2. Iran, Russia, India and Azerbaijan are the founding members of INSTC
3. INSTC is a part of Ashgabat agreement

Which of the statements given above is/are correct?


A. 1 and 2 only
B. 2 and 3 only
C. 1 and 3 only
D. None

Your Answer :
Correct Answer : D

Answer Justification :

prelims.insightsonindia.com 36
© Insights Active Learning | All rights reserved - 131815. You may not reproduce, distribute or exploit the contents in any form without
written permission by copyright owner. Copyright infringers may face civil and criminal liability
Total Marks : 200
Online Prelims TEST - 9 (SUBJECT WISE)
( InsightsIAS Mock Test Series for UPSC Preliminary Exam 2020 ) Mark Scored : 76

International North South Transport Corridor (INSTC)

• Multi modal transport corridor signed in 2000 in St. Petersburg with Iran, Russia and
India as founding members. Hence Statement 2 is incorrect.

• It was expanded to include 11 new members: Azerbaijan, Armenia, Kazakhstan, Kyrgyzstan,


Tajikistan, Turkey, Ukraine, Belarus, Oman, Syria and Bulgaria.

• It aims to link India and Iran via the sea route and then through Iran to the Caspian sea onwards
to Central Asia.

4
1 03
4 64
5
- 87
.com
a i l
g m
2 2@
am
up
s an
- da
s
Da
m
u pa
An

International North–South Transport Corridor will also synchronize with the Ashgabat
agreement, (not a part of it) a Multimodal transport agreement signed by India (2018), Oman
(2011), Iran (2011), Turkmenistan (2011), Uzbekistan (2011) and Kazakhstan (2015) (figure in the
bracket indicates the year of joining the agreement), for creating an international transport and
transit corridor facilitating transportation of goods between Central Asia and the Persian Gulf. This
route will be operationalized by mid-January 2018. Hence Statement 3 is incorrect.

https://en.wikipedia.org/wiki/International_North%E2%80%93South_Transport_Corridor

56 Consider the following statements

prelims.insightsonindia.com 37
© Insights Active Learning | All rights reserved - 131815. You may not reproduce, distribute or exploit the contents in any form without
written permission by copyright owner. Copyright infringers may face civil and criminal liability
Total Marks : 200
Online Prelims TEST - 9 (SUBJECT WISE)
( InsightsIAS Mock Test Series for UPSC Preliminary Exam 2020 ) Mark Scored : 76

1. World Energy Outlook (WEO) is published by International Energy Agency


2. India is not a member country of International Energy Forum

Which of the statements given above is/are correct?


A. 1 only
B. 2 only
C. Both 1 and 2
D. Neither 1 nor 2

Your Answer :
Correct Answer : A

Answer Justification :

The IEA flagship publication World Energy Outlook (WEO), widely regarded as the gold
standard of energy analysis. Hence Statement 1 is correct.
0 34
India hosted 16th International Energy Forum (IEF) Ministerial meet at New 1 in April, 2018.
4Delhi
6
7 54
-8
About International Energy Forum (IEF):

. c om
The International Energy Forum (IEF) aims to a il greater mutual understanding and
foster
awareness of common energy interestsgamong m its members.
2 @
2
The 72 Member Countriesu
am
ofpthe Forum are signatories to the IEF
Charter, which outlinessa n framework of the global energy dialogue
the
a
s -d
through this inter-governmental arrangement.

Da
amall six continents and accounting for around 90% of global supply and demand
Covering
p
nuoil and gas, the IEF is unique in that it comprises not only consuming and producing
Afor
countries of the International Energy Agency and OPEC.

India is a member country of International Energy Forum. Hence Statement 2 is


incorrect.

The IEF is the neutral facilitator of informal, open, informed and continuing global energy
dialogue.

The Forum's biennial Ministerial Meetings are the world's largest gathering of Energy
Ministers.

The IEF and the global energy dialogue are promoted by a permanent Secretariat of
international staff based in the Diplomatic Quarter of Riyadh, Saudi Arabia.

prelims.insightsonindia.com 38
© Insights Active Learning | All rights reserved - 131815. You may not reproduce, distribute or exploit the contents in any form without
written permission by copyright owner. Copyright infringers may face civil and criminal liability
Total Marks : 200
Online Prelims TEST - 9 (SUBJECT WISE)
( InsightsIAS Mock Test Series for UPSC Preliminary Exam 2020 ) Mark Scored : 76

https://www.ief.org/about-ief/ief-overview.aspx

https://www.ief.org/about-ief/organisation/member-countries.aspx

57 The Budapest Convention is related to which of the following matters?

A. Organic Farming
B. Ozone Depletion
C. World Trade Organization
D. Cyber Security

Your Answer : B
Correct Answer : D 4
1 03
Answer Justification : 4 64
5
87
The Convention on Cybercrime of the Council of Europe (CETS- No.185), known as the Budapest
m issue.
Convention, is the only binding international instrument onothis
i l .c
a
@ gm
It serves as a guideline for any country developing comprehensive national legislation against
2
Cybercrime and as a framework for international cooperation between State Parties to this treaty.
a m2
up
The Budapest Convention is supplemented
n
by a Protocol on Xenophobia and Racism committed
s
through computer systems. India a is not a member to the convention at present. However, recently
d a
Ministry of Home affairs- mentioned that India is reconsidering its position on becoming a member
s
Da
of the Budapest Convention because of the surge in cybercrime, especially after a push for digital

am
India.
u p
An
https://www.coe.int/en/web/cybercrime/the-budapest-convention

58 Which of the following country is not a member of Organization of the Petroleum Exporting
Countries (OPEC)?

A. Libya
B. Iran
C. Saudi Arabia
D. Qatar

Your Answer : D
Correct Answer : D

Answer Justification :

The Organization of the Petroleum Exporting Countries (OPEC) was founded in Baghdad, Iraq, with

prelims.insightsonindia.com 39
© Insights Active Learning | All rights reserved - 131815. You may not reproduce, distribute or exploit the contents in any form without
written permission by copyright owner. Copyright infringers may face civil and criminal liability
Total Marks : 200
Online Prelims TEST - 9 (SUBJECT WISE)
( InsightsIAS Mock Test Series for UPSC Preliminary Exam 2020 ) Mark Scored : 76

the signing of an agreement in September 1960 by five countries namely Islamic Republic of
Iran, Iraq, Kuwait, Saudi Arabia and Venezuela. They were to become the Founder Members of
the Organization.
These countries were later joined by Qatar (1961), Indonesia (1962), Libya (1962), the United Arab
Emirates (1967), Algeria (1969), Nigeria (1971), Ecuador (1973), Gabon (1975), Angola (2007),
Equatorial Guinea (2017) and Congo (2018). Ecuador suspended its membership in December 1992,
but rejoined OPEC in October 2007.
Indonesia suspended its membership in January 2009, reactivated it again in January 2016, but
decided to suspend its membership once more at the 171st Meeting of the OPEC Conference on 30
November 2016. Gabon terminated its membership in January 1995. However, it rejoined the
Organization in July 2016. Qatar terminated its membership on 1 January 2019.

This means that, currently, the Organization has a total of 14 Member Countries.
https://www.opec.org/opec_web/en/about_us/25.htm

4
59 Which of the following organizations publishes Travel and Tourism Competitiveness Report?
3
10
A. World Economic Forum (WEF) 4 64
5
B. The Global work and Travels Co
- 87
om
C. World Travel and Tourism Council
D. Expedia Travel Agency
i l .c
m a
g
2@
Your Answer : A
Correct Answer : A 2
am
up
Answer Justification :
s an
da
s - Competitiveness Report was first published in 2007 by the World
The Travel and Tourism
a
Economic Forum.D The report ranks selected nations according to the Travel and Tourism
m
pasub-index.
Competitiveness Index (TTCI), which scores from 1 to 6 the performance of a given country in
n u
each specific
A
https://www.weforum.org/reports/the-travel-tourism-competitiveness-report-2017

60 With reference to The National Tiger Conservation Authority, consider the following statements:
1. It is a statutory body under the Ministry of Environment, Forests and Climate Change.
2. It was constituted under provisions of the Wildlife (Protection) Act, 1972.
3. It is chaired by the Prime Minister of India.

Which of the statements given above is/are correct?


A. 1 and 2 only
B. 2 and 3 only
C. 1 and 3 only
D. 1, 2 and 3

Your Answer : A
Correct Answer : A

prelims.insightsonindia.com 40
© Insights Active Learning | All rights reserved - 131815. You may not reproduce, distribute or exploit the contents in any form without
written permission by copyright owner. Copyright infringers may face civil and criminal liability
Total Marks : 200
Online Prelims TEST - 9 (SUBJECT WISE)
( InsightsIAS Mock Test Series for UPSC Preliminary Exam 2020 ) Mark Scored : 76

Answer Justification :

The National Tiger Conservation Authority is a statutory body under the Ministry of
Environment, Forests and Climate Change constituted under provisions of the Wildlife
(Protection) Act, 1972. Hence Statement 1 and 2 is correct.

Environment Minister is the Chairman of the NTCA. Hence Statement 3 is incorrect.

Below chairman are eight experts or professionals having qualifications and experience in
wildlife conservation and welfare of people including tribals, apart from three Members of
Parliament (1 Rajya Sabha, 2 Lok Sabha).

Objectives of NTCA are

Providing statutory authority to Project Tiger so that compliance of its 3 4


directives become
1 0
legal. 4
46
Fostering accountability of Center-State in management of Tiger5Reserves, by providing a
7
basis for MoU with States within our federal structure.
-8
Providing for an oversight by Parliament. m
osurrounding
l .
Addressing livelihood interests of local people in areasc i Tiger Reserves.
m a
@g
https://projecttiger.nic.in/content/111_1_Introduction.aspx
2
m 2
61 Consider the following statements nu
pa
a
1. Indus Water Treaty was brokered sa by World Bank in 1950.
s -d
2. As per the treaty, control over three eastern rivers Ravi, Beas and Sutlej was given to Pakistan.
a
3. Ratle is the run-ofDthe-river hydroelectric power station over Jhelum River, downstream of the
m
a in Jammu and Kashmir.
village of Ratle
nup
A
Which of the statements given above is/are correct?
A. 2 only
B. 3 only
C. 1 and 3 only
D. None

Your Answer : C
Correct Answer : D

Answer Justification :

All the statements given above are incorrect.

Indus Water Treaty of 1960 brokered by the World Bank. As per the treaty, control over three
eastern rivers Ravi, Beas and Sutlej was given to India.

While control over three western rivers Indus, Jhelum and Chenab was given to Pakistan.

prelims.insightsonindia.com 41
© Insights Active Learning | All rights reserved - 131815. You may not reproduce, distribute or exploit the contents in any form without
written permission by copyright owner. Copyright infringers may face civil and criminal liability
Total Marks : 200
Online Prelims TEST - 9 (SUBJECT WISE)
( InsightsIAS Mock Test Series for UPSC Preliminary Exam 2020 ) Mark Scored : 76

Ratle (run-of-the-river hydroelectric power station) Chenab River, downstream of the village
of Ratle in Jammu and Kashmir.

62 Which of the following ideologies is closest to the “ASEAN Way”?

A. Forming formal supra-national structures and institutions


B. Settling disputes by informal cooperative methods
C. Relying on cultural rather than economic force
D. Assisting the development of Least Development Countries (LDCs)

Your Answer :
Correct Answer : B

Answer Justification :

The ASEAN Way is defined a shared of norms, principles, and values governing
0 34 the
interactions of ASEAN member states. Keeping ASEAN together despite4these 1 deep cultural,
4
economic and political differences - along with mistrust between different 6
members - has always
7 5
been an immense challenge.
- 8
m
Unlike other regional or international organizations, ASEAN
i l .copossesses its own model of diplomatic
ma
engagement: the ASEAN Way.
g
2 2@
am Indian Ocean Conference
63 Consider the following statements regarding
p
1. It has been initiated by Delhi based
n uthink tank India Foundation.
a
d as to bring
2. It is annual conference that aims together Heads of States/Governments, Ministers,

s -
thought leaders, scholars, diplomats, bureaucrats and practitioners from across the region on a
single platform.
Da
m
pastatements given above is/are correct?
Which ofuthe
A.A1nonly
B. 2 only
C. Both 1 and 2
D. Neither 1 nor 2

Your Answer :
Correct Answer : C

Answer Justification :

Both the statements given above are correct.

It has been initiated by Delhi based think tank India Foundation along with its partners from
Singapore, Sri Lanka and Bangladesh.

It is annual conference that aims to bring together Heads of States/Governments,


Ministers, thought leaders, scholars, diplomats, bureaucrats and practitioners from across

prelims.insightsonindia.com 42
© Insights Active Learning | All rights reserved - 131815. You may not reproduce, distribute or exploit the contents in any form without
written permission by copyright owner. Copyright infringers may face civil and criminal liability
Total Marks : 200
Online Prelims TEST - 9 (SUBJECT WISE)
( InsightsIAS Mock Test Series for UPSC Preliminary Exam 2020 ) Mark Scored : 76

the region on a single platform.

The 4th edition of Indian Ocean Conference (IOC) is being organised by India Foundation in
association with Government of Maldives and S. Rajaratnam School of International Studies,
Singapore on 03-04 September 2019 in Maldives. The theme of IOC 2019 is "Securing the Indian
Ocean Region: Traditional and Non-Traditional Challenges".

http://indianocean.indiafoundation.in/

64 Consider the following statements regarding Border Haats


1. They are market places organised by the two countries along the international border.
2. The border haats aim at promoting the wellbeing of the people dwelling in remote areas across the
borders of two countries.
3. Currently border haats are operational along India-Bangladesh and India-China border.

Which of the statements given above is/are correct? 4


A. 1 and 2 only 1 03
B. 2 only 4 64
5
C. 3 only
- 87
D. 2 and 3 only
.com
Your Answer : A a i l
Correct Answer : A g m
2 2@
Answer Justification : am
up
s an
da
What are Border Haats?
-
as organised by the two countries one day each week.
They are market places
D
am
p a market for buying daily commodities but also a reunion spot for families living on
It is not only
bothA nu
sides.

Aim: The border haats aim at promoting the wellbeing of the people dwelling in remote areas
across the borders of two countries, by establishing traditional system of marketing the local
produce through local markets.

The government of India and the government of Bangladesh have approved six more border haats:
two in Tripura at Palbasti and Kamalpur and four in Meghalaya at Bholaganj, Nalikata, Shibbari and
Ryngku.

India ruled out setting up trading centres along its border with China in Arunachal Pradesh saying
such an initiative could be taken only when Beijing agrees to it. Hence Statement 3 is incorrect.

https://pib.gov.in/newsite/PrintRelease.aspx?relid=155324

prelims.insightsonindia.com 43
© Insights Active Learning | All rights reserved - 131815. You may not reproduce, distribute or exploit the contents in any form without
written permission by copyright owner. Copyright infringers may face civil and criminal liability
Total Marks : 200
Online Prelims TEST - 9 (SUBJECT WISE)
( InsightsIAS Mock Test Series for UPSC Preliminary Exam 2020 ) Mark Scored : 76

65 Consider the following statements


1. Operation Samudra Maitri is a military exercise between India and Maldives
2. Mt Everest Friendship Exercise is a military exercise between Nepal and China

Which of the statements given above is/are correct?


A. 1 only
B. 2 only
C. Both 1 and 2
D. Neither 1 nor 2

Your Answer :
Correct Answer : B

Answer Justification :

Operation Samudra Maitri is a humanitarian assistance mission of India to help tsunami-hit


Indonesia. Three Indian Navy ships INS Tir, INS Sujatha and INS Shardul have 0 34 mobilised to
been
carry out Humanitarian assistance and disaster relief. Hence Statement 6 14
1
is incorrect.
7 54
The second edition of Nepal-China joint military exercise – Mt Everest - 8 Friendship Exercise-2018
also called as Sagarmatha Friendship-2018, took place in China’s
. c om southwestern Sichuan province.
Hence Statement 2 is correct.
a il
@ gm
It will focus on combating terror and disaster management training. Sagarmatha is the Nepali name
2
m2 the countries.
for Mt Everest, which stands in between both
a
n up
a
66 Consider the following statements sa regarding Nuclear Energy Agency
s
1. It is an intergovernmental- dagency that facilitates co-operation among countries with advanced
nuclear technology Dainfrastructures.
2. Country must
p amratify Non-Proliferation Treaty before joining Nuclear Energy agency.
u
An
3. It relies on member country experts to carry out much of its technical work.

Which of the statements given above is/are correct?


A. 1, 2 and 3
B. 2 and 3 only
C. 1 and 3 only
D. 1 and 2 only

Your Answer : A
Correct Answer : C

Answer Justification :

The Nuclear Energy Agency (NEA) is an intergovernmental agency that facilitates co-operation
among countries with advanced nuclear technology infrastructures to seek excellence in nuclear
safety, technology, science, environment and law. The NEA, which is under the framework of the
Organisation for Economic Co-operation and Development, is headquartered in Paris, France.

prelims.insightsonindia.com 44
© Insights Active Learning | All rights reserved - 131815. You may not reproduce, distribute or exploit the contents in any form without
written permission by copyright owner. Copyright infringers may face civil and criminal liability
Total Marks : 200
Online Prelims TEST - 9 (SUBJECT WISE)
( InsightsIAS Mock Test Series for UPSC Preliminary Exam 2020 ) Mark Scored : 76

Hence Statement 1 is correct.

A country need not to sign or ratify Non-Proliferation Treaty before joining Nuclear Energy agency.
Hence Statement 2 is incorrect.

NEA membership represents much of the world's best nuclear expertise and it maintains
strategic partnerships with key non-member countries involved in nuclear technology.

By pooling this expertise, the NEA provides each member country access to the substantial
experience of others and an opportunity to substantially leverage its resources.

The NEA establishes a climate of mutual trust and collaboration, enabling the full exchange of
experience and a frank assessment of issues.

0 34
NEA scientific and technical work is in the forefront of knowledge and is1known for its depth
and quality. 4 64
8 75
-
m
The NEA publishes consensus positions on key issues,
i l . coproviding member countries with
ma
credible references.
g
2 2@
am 3 is correct.
The NEA is cost-effective, relying on member country experts to carry out much of its
p
nu
technical work. Hence Statement
a
d as
The NEA's system
a s -of standing technical committees enables the Agency to be flexible and
responsive.D
p am
n u
A
https://www.oecd-nea.org/general/about/

67 “The costs of De-carbonization” report is published by

A. United Nations Environment Programme


B. OECD
C. World Bank
D. United Nations Framework Convention on Climate Change

Your Answer : D
Correct Answer : B

Answer Justification :

Under the Paris Agreement, OECD countries agreed to aim for a reduction of their
greenhouse gas emissions sufficient to hold the increase in the global average

prelims.insightsonindia.com 45
© Insights Active Learning | All rights reserved - 131815. You may not reproduce, distribute or exploit the contents in any form without
written permission by copyright owner. Copyright infringers may face civil and criminal liability
Total Marks : 200
Online Prelims TEST - 9 (SUBJECT WISE)
( InsightsIAS Mock Test Series for UPSC Preliminary Exam 2020 ) Mark Scored : 76

temperature to well below 2°C above pre industrial levels.

This commitment requires a massive effort to decarbonise energy and electricity generation, a
radical restructuring of the electric power sector and the rapid deployment of large amounts of low-
carbon generation technologies, in particular nuclear energy and renewable energies such as wind
and solar PV.

http://www.oecd.org/publications/the-costs-of-decarbonisation-9789264312180-en.htm

68 Which of the following country is not a member of Gulf Co-operation Council?

A. Oman
B. Qatar
C. Kuwait
D. Iran
4
Your Answer : B 1 03
Correct Answer : D 4 64
5
- 87
Answer Justification :
om
a i
Gulf Cooperation Council (is a regional intergovernmental
l.c
political and economic union
m
g except Iraq, namely: Bahrain, Kuwait,
consisting of all Arab states of the Persian Gulf
2 @Arab
2
Oman, Qatar, Saudi Arabia, and the United Emirates.
am
n up
All current member states are monarchies, including three constitutional monarchies (Qatar,
a
d as
Kuwait, and Bahrain), two absolute monarchies (Saudi Arabia and Oman), and one federal
- Emirates, which is composed of seven member states, each of which is
monarchy (the United Arab
swith
D a
an absolute monarchy its own emir). There have been discussions regarding the future
membership ofmJordan, Morocco, and Yemen.
a
nup
A

prelims.insightsonindia.com 46
© Insights Active Learning | All rights reserved - 131815. You may not reproduce, distribute or exploit the contents in any form without
written permission by copyright owner. Copyright infringers may face civil and criminal liability
Total Marks : 200
Online Prelims TEST - 9 (SUBJECT WISE)
( InsightsIAS Mock Test Series for UPSC Preliminary Exam 2020 ) Mark Scored : 76

4
1 03
4 64
5
- 87
.com
a i l
g m
2 2@
am
up
s an
- da
s
Da
m
u pa
An

69 Consider the following statements regarding International Energy Agency (IEA)


1. It is an autonomous intergovernmental organization established in the framework of the OECD
2. India is a member country of it.

Which of the statements given above is/are correct?


A. 1 only
B. 2 only
C. Both 1 and 2
D. Neither 1 nor 2

Your Answer : A
Correct Answer : A

prelims.insightsonindia.com 47
© Insights Active Learning | All rights reserved - 131815. You may not reproduce, distribute or exploit the contents in any form without
written permission by copyright owner. Copyright infringers may face civil and criminal liability
Total Marks : 200
Online Prelims TEST - 9 (SUBJECT WISE)
( InsightsIAS Mock Test Series for UPSC Preliminary Exam 2020 ) Mark Scored : 76

Answer Justification :

IEA is a Paris-based autonomous intergovernmental organization established in the framework of


the Organisation for Economic Co-operation and Development (OECD) in 1974 in the wake of
the 1973 oil crisis. Hence Statement 1 is correct.

India is not a member country of International Energy Agency. Hence Statement 2 is incorrect

The IEA acts as a policy adviser to its member states, but also works with non-member
countries, especially China, India, and Russia.

The Agency's mandate has broadened to focus on the "3Es" of effectual energy policy: energy
security, economic development, and environmental protection. The latter has focused on mitigating
climate change. The IEA has a broad role in promoting alternate energy sources (including
renewable energy), rational energy policies, and multinational energy technology co-operation.

4
70 Consider the following statements
1 03
4 64
1. Chemical Weapons Convention (CWC) is the world’s first multilateral disarmament agreement to
provide for the elimination of an entire category of weapons of mass 7 5
destruction within a fixed time
frame. -8
. c om is an intergovernmental
2. The Organisation for the Prohibition of Chemical Weapons (OPCW)
organisation and the implementing body for the Chemical
a il Weapons Convention.
3. The OPCW had been awarded Nobel Peace Prizeg inm2013 for its extensive efforts to eliminate
chemical weapons. 2 @
a m2
upis/are correct?
Which of the statements given above
n
A. 1 and 2 only
a sa
B. 1, 2 and 3
s -d
C. 1 only
D a
D. 2 and 3 m
p a only
nu :
YourAAnswer
Correct Answer : B

Answer Justification :

All the statements given above are correct.

History was made on 29 April 1997 with the entry into force of the Chemical Weapons Convention
(CWC)—the world’s first multilateral disarmament agreement to provide for the elimination of an
entire category of weapons of mass destruction within a fixed time frame.

The Organisation for the Prohibition of Chemical Weapons (OPCW) is the


implementing body for the Chemical Weapons Convention, which entered into force on 29 April
1997.

The OPCW, with its 193 Member States, oversees the global endeavor to permanently and verifiably
eliminate chemical weapons.

prelims.insightsonindia.com 48
© Insights Active Learning | All rights reserved - 131815. You may not reproduce, distribute or exploit the contents in any form without
written permission by copyright owner. Copyright infringers may face civil and criminal liability
Total Marks : 200
Online Prelims TEST - 9 (SUBJECT WISE)
( InsightsIAS Mock Test Series for UPSC Preliminary Exam 2020 ) Mark Scored : 76

The 2013 Nobel Prize for Peace was awarded to the Organisation for the Prohibition of
Chemical Weapons on 10 December 2013 for ‘its extensive efforts to eliminate chemical
weapons.

71 Which of the following countries is/are members of G-20 group


1. Argentina
2. Saudi Arabia
3. Singapore
4. Spain

Select the correct answer using the code given below


A. 1 and 3 only
B. 2 and 3 only
C. 1 and 2 only
D. 1, 2 and 3 only
4
1 03
64
Your Answer :
Correct Answer : C 5 4
- 87
Answer Justification :
.c om
i l
a The 19 countries are Argentina,
The G20 comprises 19 countries and the European Union.
g m
2@ the United Kingdom and the United States
Australia, Brazil, Canada, China, Germany, France, India, Indonesia, Italy, Japan, Mexico, Russia,
Saudi Arabia, South Africa, South Korea, 2Turkey,
m
u pa
n
a sa
s -d
Da
p am
u
An

prelims.insightsonindia.com 49
© Insights Active Learning | All rights reserved - 131815. You may not reproduce, distribute or exploit the contents in any form without
written permission by copyright owner. Copyright infringers may face civil and criminal liability
Total Marks : 200
Online Prelims TEST - 9 (SUBJECT WISE)
( InsightsIAS Mock Test Series for UPSC Preliminary Exam 2020 ) Mark Scored : 76

4
1 03
4 64
5
- 87
.com
a i l
g m
2 2@
am
up
s an
- da
s
Da
m
https://g20.org/en/summit/about/
a
nup
A
72 Consider the following statements regarding International Solar Alliance
1. The initiative was launched at the UN Climate Change Conference in Paris at the end of 2015 by the
President of France and the Prime Minister of India.
2. The primary aim of this is to provide quick and easy access to solar resource data globally.
3. Countries that do not fall within the Tropics cannot join the alliance.

Which of the statements given above is/are correct?


A. 2 and 3 only
B. 1 and 2 only
C. 1, 2 and 3
D. None

Your Answer : B
Correct Answer : B

Answer Justification :

prelims.insightsonindia.com 50
© Insights Active Learning | All rights reserved - 131815. You may not reproduce, distribute or exploit the contents in any form without
written permission by copyright owner. Copyright infringers may face civil and criminal liability
Total Marks : 200
Online Prelims TEST - 9 (SUBJECT WISE)
( InsightsIAS Mock Test Series for UPSC Preliminary Exam 2020 ) Mark Scored : 76

The initiative was launched at the UN Climate Change Conference in Paris at the end of 2015 by the
President of France and the Prime Minister of India. Hence Statement 1 is correct.

The primary aim of this is to provide quick and easy access to solar resource data globally. Hence
Statement 2 is correct.

The alliance is a treaty-based inter-governmental organization. Countries that do not fall


within the Tropics can join the alliance and enjoy all benefits as other members, with the
exception of voting rights. Hence Statement 3 is incorrect.

https://en.wikipedia.org/wiki/International_Solar_Alliance

73 Which of the following country is not an observer of SAARC?


#94000

A. Australia 4
B. France
1 03
C. European Union
4 64
5
D. China
- 87
Your Answer : C
.com
Correct Answer : B a i l
g m
2@
m2
Answer Justification :
a
States with observer status include
n up Australia, China, the European Union, Iran, Japan,
Mauritius, Myanmar, South
a saKorea and the United States.
s -d
D a
On 2 August 2006, the foreign ministers of the SAARC countries agreed in principle to grant
observer status to three applicants; the US and South Korea (both made requests in April 2006), as
am
well as thepEuropean Union (requested in July 2006). On 4 March 2007, Iran requested observer
n u
A
status, followed shortly by Mauritius.

https://en.wikipedia.org/wiki/South_Asian_Association_for_Regional_Cooperation#Observers

74 Consider the following statements regarding Treaty on the Prohibition of Nuclear Weapons
1. It includes a comprehensive set of prohibitions on participating in any nuclear weapon activities.
2. The Treaty obliges States parties to provide adequate assistance to individuals affected by the use
or testing of nuclear weapons.
3. It will enter into force 90 days after the fiftieth instrument of ratification, acceptance, approval or
accession has been deposited.

Which of the statements given above is/are correct?


A. 2 and 3 only
B. 1 and 2 only
C. 1, 2 and 3
D. None

prelims.insightsonindia.com 51
© Insights Active Learning | All rights reserved - 131815. You may not reproduce, distribute or exploit the contents in any form without
written permission by copyright owner. Copyright infringers may face civil and criminal liability
Total Marks : 200
Online Prelims TEST - 9 (SUBJECT WISE)
( InsightsIAS Mock Test Series for UPSC Preliminary Exam 2020 ) Mark Scored : 76

Your Answer : C
Correct Answer : C

Answer Justification :

All the statements given above are correct.

The Treaty on the Prohibition of Nuclear Weapons (TPNW) includes a comprehensive set
of prohibitions on participating in any nuclear weapon activities. These include undertakings
not to develop, test, produce, acquire, possess, stockpile, use or threaten to use nuclear weapons.

The Treaty also prohibits the deployment of nuclear weapons on national territory and the provision
of assistance to any State in the conduct of prohibited activities. States parties will be obliged to
prevent and suppress any activity prohibited under the TPNW undertaken by persons or on territory
under its jurisdiction or control.

The Treaty also obliges States parties to provide adequate assistance4to individuals
1 03 and appropriate
affected by the use or testing of nuclear weapons, as well as to take necessary
64 contaminated as a
measure of environmental remediation in areas under its jurisdiction or4control
result of activities related to the testing or use of nuclear weapons. 75
-8
c
The Treaty on the Prohibition of Nuclear Weapons was adopted
. om by the Conference (by a vote of 122
a il at the United Nations on 7 July 2017,
States in favour (with one vote against and one abstention)
gm
and opened for signature by the Secretary-General of the United Nations on 20 September 2017.
2 2@
It will enter into force 90 days after
a m the fiftieth instrument of ratification, acceptance,
approval or accession has beenu p
deposited.
a n
d as
https://www.un.org/disarmament/wmd/nuclear/tpnw/
a s-
D
a m
75 Consider the following statements
u p
A n
1. The European Parliament is the only legislative branch of the European Union.
2. Council of the European Union is the executive branch of the European Union, responsible for
proposing legislation, implementing decisions, upholding the EU treaties and managing the day-to-
day business of the EU.

Which of the statements given above is/are correct?


A. 1 only
B. 2 only
C. Both 1 and 2
D. Neither 1 nor 2

Your Answer : C
Correct Answer : D

Answer Justification :

Both the statements are incorrect.

prelims.insightsonindia.com 52
© Insights Active Learning | All rights reserved - 131815. You may not reproduce, distribute or exploit the contents in any form without
written permission by copyright owner. Copyright infringers may face civil and criminal liability
Total Marks : 200
Online Prelims TEST - 9 (SUBJECT WISE)
( InsightsIAS Mock Test Series for UPSC Preliminary Exam 2020 ) Mark Scored : 76

The European Parliament (EP) is the legislative branch of the European Union and one of
its seven institutions. Together with the Council of the European Union, it adopts
European legislation, normally on a proposal from the European Commission.

The Parliament is composed of 751 members (MEPs), intended to become 705 starting from the
2019–2024 legislature because of specific provisions adopted about Brexit, who represent the
second-largest democratic electorate in the world (after the Parliament of India) and the largest
trans-national democratic electorate in the world (375 million eligible voters in 2009).

The European Commission (EC) is the executive branch of the European Union,
responsible for proposing legislation, implementing decisions, upholding the EU treaties
and managing the day-to-day business of the EU.

https://en.wikipedia.org/wiki/European_Parliament

https://en.wikipedia.org/wiki/European_Commission
4
1 03
76 Consider the following pairs
4 64
5
87
Place/Issue in News Region
1. Port of Duqm Yemen -
2. Gilets Jaunes Protests France
.com
3. Farzad-B Gas Block Iran a i l
g m
2@
a
Which of the pairs given above is/arepcorrectly
m2
matched?
u
A. 3 only
s an
B. 1 and 3 only d a
C. 1 and 2 only s -
a
D. 2 and 3 onlyD
p am
u :
An
Your Answer
Correct Answer : D

Answer Justification :

Port of Duqm is located in Oman.

The yellow vests movement or yellow jackets movement is a populist, grassroots


revolutionary political movement for economic justice that began in France in October
2018. After an online petition posted in May had attracted nearly a million signatures, mass
demonstrations began on 17 November.

The Farzad B gas field is an Iranian natural gas field that was discovered in 2012. It began
production in 2013 and produces natural gas and condensates.

prelims.insightsonindia.com 53
© Insights Active Learning | All rights reserved - 131815. You may not reproduce, distribute or exploit the contents in any form without
written permission by copyright owner. Copyright infringers may face civil and criminal liability
Total Marks : 200
Online Prelims TEST - 9 (SUBJECT WISE)
( InsightsIAS Mock Test Series for UPSC Preliminary Exam 2020 ) Mark Scored : 76

77 Consider the following pairs


Issue in News Region
1. Changi Naval Base Indonesia
2. Tripoli Libya
3. Male Maldives

Which of the statements given above is/are correct?


A. 2 only
B. 2 and 3 only
C. 1 and 3 only
D. 3 only

Your Answer : B
Correct Answer : B
4
1 03
Answer Justification :
4 64
5
Changi Naval Base is located in Singapore.
- 87
om
l.c also.
Tripoli is capital of Libya. Tripoli is a city located in Lebanon
a i
Malé is the capital and most populous city in the g
m
Republic of Maldives.
2 @
m 2
a
78 Consider the following pairs
n up
Militia sometimes in the news
1. People’s Protection Unitsda
sa Turkey
Country they belong to

2. Revolutionary Guards - Iraq


3. Salwa Judum D a India
p am
Whichnu
A of the pairs given above is/are correctly matched?
A. 1 only
B. 1 and 3 only
C. 2 and 3 only
D. 3 only

Your Answer : C
Correct Answer : D

Answer Justification :

The YPG or People’s Protection Units are the armed wing of the Syrian Kurdish
Democratic Union Party.

The Islamic Revolutionary Guard Corps is a branch of Iran’s Armed Forces founded after
the 1979 Revolution by order of Ayatollah Khomeini.

prelims.insightsonindia.com 54
© Insights Active Learning | All rights reserved - 131815. You may not reproduce, distribute or exploit the contents in any form without
written permission by copyright owner. Copyright infringers may face civil and criminal liability
Total Marks : 200
Online Prelims TEST - 9 (SUBJECT WISE)
( InsightsIAS Mock Test Series for UPSC Preliminary Exam 2020 ) Mark Scored : 76

Salwa Judum was a militia that was mobilised and deployed as part of anti-insurgency
operations in Chhattisgarh, India, aimed at countering Naxalite violence in the region.

79 Consider the following statements regarding Quadrilateral Security Dialogue Summit


1. It is an informal mechanism between India, the US, Japan and South Korea
2. It is interpreted as a joint effort to counter China’s influence in the Indo-Pacific region.

Which of the statements given above is/are correct?


A. 1 only
B. 2 only
C. Both 1 and 2
D. Neither 1 nor 2

Your Answer : C
Correct Answer : B
4
1 03
Answer Justification :
64
7 Japan. Hence Statement 54
It is an informal mechanism between India, the US, Australia8and
-
1 is incorrect m
i l .co
Hence Statement 2 is correct. g ma influence in the Indo-Pacific region.
It is interpreted as a joint effort to counter China’s

2 2@
m
pa attributed to Japanese Prime Minister Shinzo Abe.
The idea of the Quad could be originally
u
n
sa
https://foreignpolicy.com/2018/07/23/india-is-the-weakest-link-in-the-quad/
a
s -d
Dastatements regarding United Nations Human Rights Council
80 Consider the following
p am elected for staggered three-year terms on a regional group basis.
1. It has 47 members
u
Anestablished by the UN General Assembly to replace the UN Commission on Human Rights.
2. It was

Which of the statements given above is/are correct?


A. 1 only
B. 2 only
C. Both 1 and 2
D. Neither 1 nor 2

Your Answer :
Correct Answer : C

Answer Justification :

Both the statements are correct.

The United Nations Human Rights Council is a United Nations body whose mission is to
promote and protect human rights around the world.

prelims.insightsonindia.com 55
© Insights Active Learning | All rights reserved - 131815. You may not reproduce, distribute or exploit the contents in any form without
written permission by copyright owner. Copyright infringers may face civil and criminal liability
Total Marks : 200
Online Prelims TEST - 9 (SUBJECT WISE)
( InsightsIAS Mock Test Series for UPSC Preliminary Exam 2020 ) Mark Scored : 76

The UNHRC has 47 members elected for staggered three-year terms on a regional group
basis. The headquarters of UNHRC is in Geneva, Switzerland.

The UNHRC investigates allegations of breaches of human rights in UN member states, and
addresses important thematic human rights issues such as freedom of association and assembly,
freedom of expression, freedom of belief and religion, women's rights, LGBT rights, and the rights
of racial and ethnic minorities.

The UNHRC was established by the UN General Assembly on March 15, 2006 (by
resolution A/RES/60/251) to replace the UN Commission on Human Rights (UNCHR,
herein CHR) that had been strongly criticised for allowing countries with poor human rights
records to be members

https://en.wikipedia.org/wiki/United_Nations_Human_Rights_Council

81 Consider the following statements regarding TRAFFIC


0 34
41to the conservation of
1. It is a joint program of WWF and UNEP
6
2. TRAFFIC works to ensure that trade in wild plants and animals is not a threat
4
nature. 75
-8
Which of the statements given above is/are correct?
. c om
A. 1 only a il
B. 2 only
@ gm
C. Both 1 and 2 2 2
D. Neither 1 nor 2 am p
u
s an
Your Answer : C a
Correct Answer : B
s -d
Da
am
Answer Justification :
n up
A the wildlife trade monitoring network, is a joint program of WWF and IUCN –
TRAFFIC,
the International Union for Conservation of Nature. Hence Statement 1 is incorrect

TRAFFIC works to ensure that trade in wild plants and animals is not a threat to the
conservation of nature. TRAFFIC has gained its greatest reputation from supporting CITES, the
Convention on International Trade in Endangered Species. Since TRAFFIC was created in 1976 it
has helped with the evolution of this international wildlife trade treaty. TRAFFIC North America
(NA) works through its offices in the United States, Canada and Mexico, and with its programs in
Central America and the Caribbean. Hence Statement 2 is correct.

TRAFFIC focuses on leveraging resources, expertise and awareness of the latest globally urgent
species trade issues such as tiger parts, elephant ivory and rhino horn. Large scale commercial
trade in commodities like timber and fisheries products are also addressed and linked to work on
developing rapid results and policy improvements.

https://www.worldwildlife.org/initiatives/traffic-the-wildlife-trade-monitoring-network

prelims.insightsonindia.com 56
© Insights Active Learning | All rights reserved - 131815. You may not reproduce, distribute or exploit the contents in any form without
written permission by copyright owner. Copyright infringers may face civil and criminal liability
Total Marks : 200
Online Prelims TEST - 9 (SUBJECT WISE)
( InsightsIAS Mock Test Series for UPSC Preliminary Exam 2020 ) Mark Scored : 76

82 Consider the following statements regarding National Policy on Bio-fuels


1. The Policy encourages setting up of supply chain mechanisms for biodiesel production from non-
edible oilseeds.
2. Policy prohibits the use of surplus food grains for production of ethanol for blending with petrol
citing food security.
3. Policy indicates a viability gap funding scheme ethanol Bio refineries.

Which of the statements given above is/are correct?


A. 1 and 3 only
B. 2 and 3 only
C. 1 only
D. 1, 2 and 3

Your Answer :
Correct Answer : A
4
Answer Justification : 1 03
4 64
5
87
Salient Features of National Policy on Biofuels – 2018
-
om
.c First Generation (1G) bioethanol &
1. The Policy categorises biofuels as "Basic Biofuels"ilviz.

g ma
biodiesel and "Advanced Biofuels" - Second Generation (2G) ethanol, Municipal Solid Waste
(MSW) to drop-in fuels, Third Generation@(3G) biofuels, bio-CNG etc. to enable extension of
22 under each category.
appropriate financial and fiscal incentives
m
u pa
a n
2. The Policy expands the
d asscope of raw material for ethanol production by allowing use of
Sugarcane Juice, -
s Sugar containing materials like Sugar Beet, Sweet Sorghum, Starch
D a
containing materials like Corn, Cassava, Damaged food grains like wheat, broken rice, Rotten
m
a unfit for human consumption for ethanol production.
Potatoes,
nup
A
3. Farmers are at a risk of not getting appropriate price for their produce during the surplus
production phase. Taking this into account, the Policy allows use of surplus food
grains for production of ethanol for blending with petrol with the approval of
National Biofuel Coordination Committee. Hence Statement 2 is incorrect.

4. With a thrust on Advanced Biofuels, the Policy indicates a viability gap funding scheme for 2G
ethanol Bio refineries of Rs.5000 crore in 6 years in addition to additional tax incentives,
higher purchase price as compared to 1G biofuels.

5. The Policy encourages setting up of supply chain mechanisms for biodiesel production from
non-edible oilseeds, Used Cooking Oil, short gestation crops.

6. Roles and responsibilities of all the concerned Ministries/Departments with respect to biofuels
has been captured in the Policy document to synergise efforts.

prelims.insightsonindia.com 57
© Insights Active Learning | All rights reserved - 131815. You may not reproduce, distribute or exploit the contents in any form without
written permission by copyright owner. Copyright infringers may face civil and criminal liability
Total Marks : 200
Online Prelims TEST - 9 (SUBJECT WISE)
( InsightsIAS Mock Test Series for UPSC Preliminary Exam 2020 ) Mark Scored : 76

https://pib.gov.in/Pressreleaseshare.aspx?PRID=1532265

83 Consider the following statements


1. Krishna River originates at Mahabaleshwar in Maharashtra.
2. Krishna Water Dispute Tribunal was first setup in 1969.
3. Krishna River joins Bay of Bengal near Chennai

Which of the statements given above is/are correct?


A. 1, 2 and 3
B. 2 and 3 only
C. 1 only
D. 1 and 2 only

Your Answer :
Correct Answer : D
4
1 03
64
Answer Justification :
54
The Krishna is an east-flowing river that originates at Mahabaleshwar
- 87 in Maharashtra and merges
with the Bay of Bengal, flowing through Maharashtra, Karnataka,
o m Telangana and Andhra Pradesh.
l c
. 33% of the total area of the four
Together with its tributaries, it forms a vast basin that covers
i
states. Hence Statement 1 is correct. a
@ gm
2
In 1969, the Krishna Water Disputes Tribunal2 (KWDT) was set up under the Inter-State River Water
am in 1973. The report, which was published in 1976,
Dispute Act, 1956, and presented its report
p
u
divided the 2060 TMC (thousandnmillion cubic feet) of Krishna water at 75 per cent dependability
a
sMaharashtra,
a
into three parts: 560 TMC for
d 2 is correct. 700 TMC for Karnataka and 800 TMC for Andhra
Pradesh. Hence Statement
s -
Da
Krishna River
a mempties into the Bay of Bengal at Hamasaladeevi (near Koduru) in Andhra
p
Pradesh.uHence Statement 3 is incorrect.
An
https://indianexpress.com/article/explained/explained-what-is-the-krishna-water-dispute-and-who-all
-are-involved-5975736/

84 Consider the following statements regarding United Nations Convention to Combat Desertification
(UNCCD)
1. UNCCD was adopted in 1994 and ratified by nearly 100 countries
2. It is committed to a bottom-up approach, encouraging the participation of local people in combating
desertification and land degradation.
3. India takes over COP Presidency from China for next two years

Which of the statements given above is/are correct?


A. 1 and 2 only
B. 2 only
C. 1 only
D. 2 and 3 only

prelims.insightsonindia.com 58
© Insights Active Learning | All rights reserved - 131815. You may not reproduce, distribute or exploit the contents in any form without
written permission by copyright owner. Copyright infringers may face civil and criminal liability
Total Marks : 200
Online Prelims TEST - 9 (SUBJECT WISE)
( InsightsIAS Mock Test Series for UPSC Preliminary Exam 2020 ) Mark Scored : 76

Your Answer : D
Correct Answer : D

Answer Justification :

UNCCD was adopted in Paris on 17 June 1994 and ratified by 196 countries & European
Union. India ratified the UNCCD Convention on December 1996. Hence Statement 1 is
incorrect.

It is committed to a bottom-up approach, encouraging the participation of local people in combating


desertification and land degradation. Hence Statement 2 is correct.

India takes over COP Presidency of UNCCD from China for next two years. Through hosting COP
14, India will highlight its leadership in navigating the land management agenda at global level. It
will also provide a stage to mainstream sustainable land management in country’s national
development policies. Hence Statement 3 is correct.
4
https://www.pib.nic.in/PressReleasePage.aspx?PRID=1583898 1 03
4 64
5
85 Consider the following statements regarding Bombay blood group - 87
. c om H antigen.
1. Individuals with the rare Bombay phenotype (hh) do not express
2. It is mostly found in South Asia and South East Asia. il
m a
2
Which of the statements given above is/are correct? @g
m 2
pa
A. 1 only
B. 2 only
n u
C. Both 1 and 2
a sa
D. Neither 1 nor 2 - d
s
Your Answer :m
Da
Correct u pa : A
Answer
An
Answer Justification :

The h/h blood group, also known as Oh or the Bombay blood group, is a rare blood type. This blood
phenotype was first discovered in Bombay, now known as Mumbai, in India, by Dr. Y. M. Bhende in
1952. It is mostly found in South Asia (India, Bangladesh, Pakistan) and parts of Middle East such
as Iran. Hence Statement 1 is correct.

Individuals with the rare Bombay phenotype (hh) do not express H antigen (also called substance
H), the antigen which is present in blood group O. As a result, they cannot make A antigen (also
called substance A) or B antigen (substance B) on their red blood cells, whatever alleles they may
have of the A and B blood-group genes, because A antigen and B antigen are made from H antigen.
Hence Statement 2 is incorrect.

https://en.wikipedia.org/wiki/Hh_blood_group

https://indianexpress.com/article/explained/what-is-bombay-blood-group-rare-and-sought-after-5984

prelims.insightsonindia.com 59
© Insights Active Learning | All rights reserved - 131815. You may not reproduce, distribute or exploit the contents in any form without
written permission by copyright owner. Copyright infringers may face civil and criminal liability
Total Marks : 200
Online Prelims TEST - 9 (SUBJECT WISE)
( InsightsIAS Mock Test Series for UPSC Preliminary Exam 2020 ) Mark Scored : 76

653/

86 Which of the following tribes is/are present in Ladakh Region


1. Balti
2. Changpa
3. Purigpa

Select the correct answer using the code given below


A. 1 and 2 only
B. 2 only
C. 2 and 3 only
D. 1, 2 and 3

Your Answer : D
Correct Answer : D
4
1 03
64
Answer Justification :

The total tribal population in Ladakh region is more than 97%. The8
region
54
7 is inhabited by following
-
Scheduled Tribes, namely: m o
i l .c
Balti a
Beda
@ gm
Bot, Boto 2 2
Brokpa, Drokpa, Dard, Shin
pam
Changpa n u
Garra a sa
Mon
s -d
Purigpa Da
a m
u p
https://www.insightsonindia.com/2019/09/12/insights-daily-current-affairs-pib-12-september-2019/
An
87 Consider the following statements regarding Government e Marketplace (GeM)
1. It is a one stop portal to facilitate online procurement of common use Goods & Services required by
various Government Departments.
2. Only Central government Ministries/Departments including its attached/subordinate offices are
eligible to procure goods from its platform

Which of the statements given above is/are correct?


A. 1 only
B. 2 only
C. Both 1 and 2
D. Neither 1 nor 2

Your Answer : A
Correct Answer : A

prelims.insightsonindia.com 60
© Insights Active Learning | All rights reserved - 131815. You may not reproduce, distribute or exploit the contents in any form without
written permission by copyright owner. Copyright infringers may face civil and criminal liability
Total Marks : 200
Online Prelims TEST - 9 (SUBJECT WISE)
( InsightsIAS Mock Test Series for UPSC Preliminary Exam 2020 ) Mark Scored : 76

Answer Justification :

Government e-Marketplace (GeM) is a one stop portal to facilitate online procurement of common
use Goods & Services required by various Government Departments / Organizations / PSUs. GeM
aims to enhance transparency, efficiency and speed in public procurement.

It provides the tools of e-bidding, reverse e-auction and demand aggregation to facilitate the
government users achieve the best value for their money. Hence Statement 1 is correct.

Who can buy/purchase through GeM?

All Central government and State Government Ministries/Departments including its


attached/subordinate offices, Central and State autonomous bodies, Central and State Public Sector
Units and local bodies etc. are authorized to make procurement through GeM portal. Hence
Statement 2 is incorrect.

4
http://vikaspedia.in/e-governance/online-citizen-services/government-to-business-services-g2b/gover
nment-e-marketplace 03
6 41
4
75
88 Consider the following statements regarding Eurasian economic union -8
. c om
1. It is an international organization for regional economic integration.
2. Russia, Belarus, Armenia and China are members of it. il
a
@ gm
2
Which of the statements given above is/are correct?
2
A. 1 only m
B. 2 only u pa
n
C. Both 1 and 2
a sa
D. Neither 1 nor 2
s -d
a
Your Answer : D
p am: A
Correct Answer
u
An
Answer Justification :

About Eurasian economic union:

It is an international organization for regional economic integration.


It has international legal personality and is established by the Treaty on the Eurasian
Economic Union.
Composition: Includes Russia, Belarus, Armenia, Kyrgyzstan and Kazakhstan. China
is not a member.
It has free movement of goods, services and labour.
It has its own bureaucratic structure.

prelims.insightsonindia.com 61
© Insights Active Learning | All rights reserved - 131815. You may not reproduce, distribute or exploit the contents in any form without
written permission by copyright owner. Copyright infringers may face civil and criminal liability
Total Marks : 200
Online Prelims TEST - 9 (SUBJECT WISE)
( InsightsIAS Mock Test Series for UPSC Preliminary Exam 2020 ) Mark Scored : 76

4
1 03
4 64
5
- 87
.com
a i l
g m
2@
a m2
n up
https://www.insightsonindia.com/2019/09/12/insights-daily-current-affairs-pib-12-september-2019/
a sa
s -d
89 Consider the followingastatements regarding ASEAN.
D by an organization formed in 31 July 1961 called the Association of Southeast
m
1. ASEAN was preceded
u
Asia (ASA),paagroup consisting of the Philippines, the Federation of Malaya, and Thailand.
An is not a member of ASEAN.
2. Vietnam
3. ASEAN established Asian Development Bank for the socio-economic development of Asia and
Pacific.

Which of the statements given above is/are correct?


A. 1 and 3 only
B. 2 and 3 only
C. 1 only
D. None

Your Answer : A
Correct Answer : C

Answer Justification :

The Association of Southeast Asian Nations is a regional intergovernmental organization comprising


ten countries in Southeast Asia, which promotes intergovernmental cooperation and facilitates

prelims.insightsonindia.com 62
© Insights Active Learning | All rights reserved - 131815. You may not reproduce, distribute or exploit the contents in any form without
written permission by copyright owner. Copyright infringers may face civil and criminal liability
Total Marks : 200
Online Prelims TEST - 9 (SUBJECT WISE)
( InsightsIAS Mock Test Series for UPSC Preliminary Exam 2020 ) Mark Scored : 76

economic, political, security, military, educational, and sociocultural integration among its members
and other countries in Asia. Hence Statement 1 is correct.

ASEAN was preceded by an organisation formed on 31 July 1961 called the Association of Southeast
Asia (ASA), a group consisting of Thailand, the Philippines, and the Federation of Malaya. ASEAN
itself was created on 8 August 1967, when the foreign ministers of five countries: Indonesia,
Malaysia, the Philippines, Singapore, and Thailand, signed the ASEAN Declaration.

Vietnam is a member of ASEAN. Hence Statement 2 is incorrect.

The Asian Development Bank (ADB) is a regional development bank established on 19 December
1966. It is not established by ASEAN. Hence Statement 3 is incorrect.

https://asean.org/asean/about-asean/

90 Consider the following statements regarding International Tribunal for the Law of the Sea (ITLOS)
34 arising
1. ITLOS is an independent judicial body established by the UNCLOS to adjudicate0disputes
out of the interpretation and application of the UNCLOS Convention. 6 41
4
2. The Tribunal is open only to the states parties to the convention. 75
-8
Which of the statements given above is/are correct?
. c om
A. 1 only a il
B. 2 only
@ gm
C. Both 1 and 2
2 2
D. Neither 1 nor 2 m
u pa
a n
Your Answer : s
Correct Answer : A - da
s
Da
m
Answer Justification :
u pa
An
International Tribunal for the Law of the Sea is an independent judicial body established by the
UNCLOS to adjudicate disputes arising out of the interpretation and application of the UNCLOS
Convention. Hence Statement 1 is correct.

HQ - Hamburg, Germany.

The Tribunal is open to States Parties to the Convention.

It is also open to entities other than States Parties, i.e., States or intergovernmental
organisations which are not parties to the Convention and to state enterprises and private entities.
Hence Statement 2 is incorrect.

Recently, Dr. Neeru Chadha has become the first Indian woman to be elected as a judge at the
International Tribunal for the Law of the Seas (ITLOS).

https://www.itlos.org/the-tribunal/

prelims.insightsonindia.com 63
© Insights Active Learning | All rights reserved - 131815. You may not reproduce, distribute or exploit the contents in any form without
written permission by copyright owner. Copyright infringers may face civil and criminal liability
Total Marks : 200
Online Prelims TEST - 9 (SUBJECT WISE)
( InsightsIAS Mock Test Series for UPSC Preliminary Exam 2020 ) Mark Scored : 76

91 Consider the following statements regarding Fall Armyworm


1. It is an invasive species that is native to tropical and subtropical regions of the Americas.
2. It significantly impacted the productivity of coffee plantation in Western Ghats region.

Which of the statements given above is/are correct?


A. 1 only
B. 2 only
C. Both 1 and 2
D. Neither 1 nor 2

Your Answer :
Correct Answer : A

Answer Justification :

What is Fall Armyworm (FAW)?


0 34
1
4Statement
It is a native of the tropical and sub-tropical regions of the Americas. Hence
4 6 1 is
correct.
8 75
-
First detected in the African continent in 2016. Since then, it m has spread to other countries such as
China, Thailand, Malaysia and Sri Lanka. i l .co
g ma
2 @
In the absence of natural control or good management, it can cause significant damage to crops. It
prefers maize, but can feed on more than 80
a m2 additional species of crops, including rice, sorghum,
millet, sugarcane, vegetable crops and
u p cotton. There is no substantial evidence of its impact on
nGhats Region. Hence Statement 2 is incorrect.
sa
Coffee Plantation in Western
a
s -d
92 Which of the followingD acountries have signed The Joint Comprehensive Plan of Action with Iran in
2015?
p am
1. U.S.A u
2. U.K A
n
3. Germany
4. China

Select the correct answer using the code given below


A. 1 and 4 only
B. 1 and 2 only
C. 1. 2, 3 and 4
D. 2, 3 and 4 only

Your Answer :
Correct Answer : C

Answer Justification :

On July 14, 2015, the P5+1 (China, France, Germany, Russia, the United Kingdom, and the
United States), the European Union (EU), and Iran reached a Joint Comprehensive Plan of

prelims.insightsonindia.com 64
© Insights Active Learning | All rights reserved - 131815. You may not reproduce, distribute or exploit the contents in any form without
written permission by copyright owner. Copyright infringers may face civil and criminal liability
Total Marks : 200
Online Prelims TEST - 9 (SUBJECT WISE)
( InsightsIAS Mock Test Series for UPSC Preliminary Exam 2020 ) Mark Scored : 76

Action (JCPOA) to ensure that Iran’s nuclear program will be exclusively peaceful.

https://www.armscontrol.org/factsheets/JCPOA-at-a-glance

93 Consider the following statements regarding Indian ports


1. Marmagao Port, situated at the entrance of the Zuari estuary, is a natural harbour in Goa.
2. Visakhapatnam Port in Andhra Pradesh is a land-locked harbour
3. The Kamarajar Port Limited is a corporatized major port and is registered as a public company.

Which of the statements given above is/are correct?


A. 1 only
B. 2 and 3 only
C. 1 and 2 only
D. 1, 2 and 3

4
03
Your Answer : D
Correct Answer : D 1
4 64
5
Answer Justification :
- 87
All the statements given above are correct. .com
a i l
Marmagao Port, situated at the entrance of the
@ gmZuari estuary, is a natural harbour in Goa.
It gained significance after its remodelling2in21961 to handle iron-ore exports to Japan. Construction
am the hinterland of this port. Karnataka, Goa, Southern
of Konkan railway has considerably extended
p
n u
Maharashtra constitute its hinterland.
a sa
Visakhapatnam Port in
s - dAndhra Pradesh is a land-locked harbour, connected to the sea by a
channel cut throughasolid rock and sand. An outer harbour has been developed for handling iron-
D
ore, petroleummand general cargo. Andhra Pradesh and Telangana are the main hinterland for this
port. u pa
An
Ennore Port, officially renamed Kamarajar Port Limited, is located on the Coromandel
Coast about 24 km north of Chennai Port, Chennai, it is the 12th major port of India, and the first
port in India which is a public company.

94 Which of the following Indian cities are a part of UNESCO's Creative Cities Network?
1. Jaipur
2. Mysore
3. Varanasi
4. Chennai

Select the correct answer using the code given below.


A. 1 and 3 only
B. 1, 3 and 4 only
C. 2, 3 and 4 only
D. 1, 2, 3 and 4

prelims.insightsonindia.com 65
© Insights Active Learning | All rights reserved - 131815. You may not reproduce, distribute or exploit the contents in any form without
written permission by copyright owner. Copyright infringers may face civil and criminal liability
Total Marks : 200
Online Prelims TEST - 9 (SUBJECT WISE)
( InsightsIAS Mock Test Series for UPSC Preliminary Exam 2020 ) Mark Scored : 76

Your Answer :
Correct Answer : B

Answer Justification :

The UNESCO Creative Cities Network (UCCN) was created in 2004 to promote cooperation with
and among cities that have identified creativity as a strategic factor for sustainable urban
development.

The Network covers seven creative fields: Crafts and Folk Arts, Media Arts, Film, Design,
Gastronomy, Literature and Music.

The Creative Cities Network is a privileged partner of UNESCO, not only as a platform for reflection
on the role of creativity as a lever for sustainable development but also as a breeding ground of
action and innovation, notably for the implementation of the 2030 Agenda for Sustainable
Development.
34
Two Indian cities for first time have been designated as members of UNESCO's10Creative Cities
Network (UCCN). 1. Varanasi City (Uttar Pradesh): It has been added4in 64the City of Music
75 in the City of Crafts and
category of network and 2. Jaipur City (Rajasthan): It has been added
8
Folk Art category of network. -
. c om
Chennai has been included in the UNESCO Creative a il Cities Network for its rich musical
tradition. Including Chennai, a total of 64 cities g m 44 countries have joined the UNESCO
from
Creative Cities Network. 2 @
m 2
u pa
https://en.unesco.org/about-us/introducing-unesco
n
a sa
s - d Platform’, a first of its kind, unified access portal which brings
95 The ‘Women Entrepreneurship
together women fromDadifferent parts of India to realize their entrepreneurial aspirations, is an
initiative of am
n up
A.ASIDBI (Small Industries Development Bank of India)
B. Ministry of Women and Child Development
C. FICCI (Federation of Indian Chambers of Commerce and Industry)
D. NITI Aayog

Your Answer : D
Correct Answer : D

Answer Justification :

The Women Entrepreneurship Platform (WEP)

NITI Aayog has launched a Women Entrepreneurship Platform (WEP) for providing an ecosystem
for budding & existing women entrepreneurs across the country. SIDBI has partnered with NITI
Aayog to assist in this initiative.

As an enabling platform, WEP is built on three pillars- Iccha Shakti, Gyaan Shakti & Karma Shakti

prelims.insightsonindia.com 66
© Insights Active Learning | All rights reserved - 131815. You may not reproduce, distribute or exploit the contents in any form without
written permission by copyright owner. Copyright infringers may face civil and criminal liability
Total Marks : 200
Online Prelims TEST - 9 (SUBJECT WISE)
( InsightsIAS Mock Test Series for UPSC Preliminary Exam 2020 ) Mark Scored : 76

Iccha Shakti represents motivating aspiring entrepreneurs to start their business.


Gyaan Shakti represents providing knowledge and ecosystem support to women
entrepreneurs to help them foster entrepreneurship.
Karma Shakti represents providing hands-on support to entrepreneurs in setting-up and
scaling up businesses.

https://www.startupindia.gov.in/content/sih/en/government-schemes/Wep.html

96 Consider the following statements


1. Sangai is the state animal of Mizoram and largely found in Keibul Lamjao National Park
2. Leptospirosis is a viral infection in rodents and other wild and domesticated species, and humans.

Which of the statements given above is/are correct?


A. 1 only
B. 2 only
4
03
C. Both 1 and 2
D. Neither 1 nor 2 1
4 64
5
Your Answer :
- 87
om
Correct Answer : D

i l .c
Answer Justification :
m a
g
2@
m2
Both the statements are incorrect.
a
n up and largely found in Keibul Lamjao National Park.
Sangai is the state animal of Manipur
a sa
- d infection in rodents and other wild and domesticated species, and
Leptospirosis is a bacterial
s
humans.
Da
p am
u
97 Which ofnthe
A following is/are condition(s) for recognizing a party as National Party?
1. If it wins two per cent of seats in the Lok Sabha at a general election; and these candidates are
elected from three states.
2. If it is recognized as a state party in four states.
3. If it wins three per cent of seats in the legislative assembly at a general election to the legislative
assembly of the state concerned or 3 seats in the assembly, whichever is more.

Which of the statements given above is/are correct?


A. 1 and 2 only
B. 2 only
C. 3 only
D. 1, 2 and 3

Your Answer : D
Correct Answer : A

Answer Justification :

prelims.insightsonindia.com 67
© Insights Active Learning | All rights reserved - 131815. You may not reproduce, distribute or exploit the contents in any form without
written permission by copyright owner. Copyright infringers may face civil and criminal liability
Total Marks : 200
Online Prelims TEST - 9 (SUBJECT WISE)
( InsightsIAS Mock Test Series for UPSC Preliminary Exam 2020 ) Mark Scored : 76

Party is recognised as a national party if any of the following conditions is fulfilled:

1. If it secures six per cent of valid votes polled in any four or more states at a general election to
the Lok Sabha or to the legislative assembly; and, in addition, it wins four seats in the Lok Sabha
from any state or states; or

2. If it wins two per cent of seats in the Lok Sabha at a general election; and these candidates are
elected from three states; or

3. If it is recognised as a state party in four states

A party is recognised as a state party in a state if any of the following conditions is


fulfilled:

0 34
1
4election
4 6
1. If it secures six per cent of the valid votes polled in the state at a general to the
5
legislative assembly of the state concerned; and, in addition, it wins 27seats in the assembly of the
state concerned; or -8
. c om
il state at a general election to the Lok
2. If it secures six per cent of the valid votes polled in the
a
Sabha from the state concerned; and, in addition, itmwins 1 seat in the Lok Sabha from the state
concerned; or 2 @g
a m2
3. If it wins three per cent of seats inpthe legislative assembly at a general election to the legislative
assembly of the state concerned a noru3 seats in the assembly, whichever is more; or
d as
s - Lok Sabha for every 25 seats or any fraction thereof allotted to the state at
4. If it wins 1 seat in the
a
a general electionDto the Lok Sabha from the state concerned; or
p am
5. If it nu eight per cent of the total valid votes polled in the state at a General Election to the
secures
A
Lok Sabha from the state or to the legislative assembly of the state. This condition was added in
2011.

98 Consider the following statements


1. Project CHAMAN is a pioneer project aims for the strategic development of the fisheries sector.
2. Mission Fingerling is a programme aimed at achieving holistic development of horticulture sector.

Which of the statements given above is/are correct?


A. 1 only
B. 2 only
C. Both 1 and 2
D. Neither 1 and 2

Your Answer :
Correct Answer : D

prelims.insightsonindia.com 68
© Insights Active Learning | All rights reserved - 131815. You may not reproduce, distribute or exploit the contents in any form without
written permission by copyright owner. Copyright infringers may face civil and criminal liability
Total Marks : 200
Online Prelims TEST - 9 (SUBJECT WISE)
( InsightsIAS Mock Test Series for UPSC Preliminary Exam 2020 ) Mark Scored : 76

Answer Justification :

Both the statements are incorrect.

Project CHAMAN is a pioneer project to provide strategic development to the horticulture


sector, so as to increase farmer’s income. This project is being implemented by National Crop
Forecast Centre (MNCFC) using remote sensing technology and is likely to be completed in March
2018 in all the states.

Mission Fingerling, a programme aimed at achieving the Blue Revolution by enabling


holistic development and management of fisheries.

99 Okjokull glacier, recently seen in news is located in

A. Canada
B. Greenland 4
C. Antarctica 1 03
D. Iceland
4 64
5
Your Answer : - 87
Correct Answer : D
.com
a i l
Answer Justification : g m
2 2@
m glacier lost due to climate change.
a
The Okjokull glacier became the first Icelandic
n up
sa
https://indianexpress.com/article/trending/trending-globally/memorial-plaque-installed-at-okjokull-gl
a
d
acier-the-first-one-in-iceland-lost-to-climate-change-5917394/
-
s
Da
m
100 Consider theafollowing statements
u p
n is closer to Kolkata than it is to Chennai.
1. Port Blair
A
2. Duncan passage separates Middle Andaman from South Andaman.

Which of the statements given above is/are correct?


A. 1 only
B. 2 only
C. Both 1 and 2
D. Neither 1 nor 2

Your Answer : C
Correct Answer : A

Answer Justification :

prelims.insightsonindia.com 69
© Insights Active Learning | All rights reserved - 131815. You may not reproduce, distribute or exploit the contents in any form without
written permission by copyright owner. Copyright infringers may face civil and criminal liability
Total Marks : 200
Online Prelims TEST - 9 (SUBJECT WISE)
( InsightsIAS Mock Test Series for UPSC Preliminary Exam 2020 ) Mark Scored : 76

Duncan pass is located between South and little Andaman. It is a strait in the Indian Ocean. It is
about 48 km wide; it separates Rutland Island to the north, and Little Andaman to the south. West
of Duncan Passage is the Bay of Bengal; east is the Andaman Sea. Several small islands and islets
lie along the passage.

4
1 03
4 64
5
- 87
.com
a i l
g m
2 2@
am
up
s an
- da
s
Da
m
u pa
An

prelims.insightsonindia.com 70
© Insights Active Learning | All rights reserved - 131815. You may not reproduce, distribute or exploit the contents in any form without
written permission by copyright owner. Copyright infringers may face civil and criminal liability

You might also like